Pathophysiology test 1

¡Supera tus tareas y exámenes ahora con Quizwiz!

10. As part of a screening program for prostate cancer, men at a senior citizens' center are having their blood levels of prostate-specific antigen (PSA) measured. Which of the following statements would best characterize a high positive predictive value but a low negative predictive value for this screening test? A) All of the men who had high PSA levels developed prostate cancer; several men who had low PSA levels also developed prostate cancer. B) All of the men who had low PSA levels were cancer-free; several men who had high levels also remained free of prostate cancer. C) Men who had low PSA levels also displayed false-positive results for prostate cancer; men with high levels were often falsely diagnosed with prostate cancer. D) The test displayed low sensitivity but high specificity.

A) All of the men who had high PSA levels developed prostate cancer; several men who had low PSA levels also developed prostate cancer.

7. The emergency department is awaiting the arrival of a spinal cord-injured patient. Knowing the innervation of the diaphragm, a patient with which type of injury may be in need of immediate mechanical ventilation? Injury to the A) C4 area. B) C7 area. C) T1 area. D) T4 area.

A) C4 area.

2. A 21-year-old male client has suffered a head injury during a crash on his motorcycle, and a deficit that assessments have revealed is an impaired swallowing mechanism. He has also developed aspiration pneumonia. Which of the following statements most accurately captures an aspect of his condition? A) His vocal folds are likely not performing their normal function. B) His epiglottis is covering his larynx. C) His vocal folds have been compromised. D) His tracheobronchial tree is intermittently obstructed.

A) His vocal folds are likely not performing their normal function.

16. As part of a community class, student nurses are developing a class to teach expectant parents the importance of having their child properly secured in a child safety seat. During the class, the students are going to have a safety officer examine the car seats that the parents have installed in their vehicles. This is an example of which type of prevention? A) Primary prevention B) Secondary prevention C) Tertiary prevention D) Prognosis enhancement

A) Primary prevention

16. Following a winter power outage, a client who had been using a home gasoline generator began to experience dizziness and headaches and was diagnosed with carbon monoxide poisoning. What is the goal of hyperbaric oxygen treatment for carbon monoxide poisoning? A) To increase the amount of oxygen carried in the dissolved state B) To increase the production of unbound hemoglobin C) To stimulate the release of oxygen at the capillaries D) To remove bound CO from hemoglobin

A) To increase the amount of oxygen carried in the dissolved state

15. A 44-year-old woman has developed calf pain during a transatlantic flight. She is extremely short of breath upon arrival at her destination. She was subsequently diagnosed with a pulmonary embolism (PE) that resolved with anticoagulant therapy. Which of these statements best characterizes the underlying problem of her PE? A) Ventilation was occurring, but perfusion was inadequate causing shortness of breath. B) The combination of normal perfusion but compromised ventilation caused hypoxia. C) She developed a transient anatomic shunt resulting in impaired oxygenation. D) Impaired gas diffusion across alveolar membranes resulted in dyspnea and hypoxia.

A) Ventilation was occurring, but perfusion was inadequate causing shortness of breath.

11. A 60-year-old male hospital patient with a diagnosis of chronic obstructive pulmonary disease (COPD) is undergoing lung function tests to gauge the progression of his disease. Which of the following aspects of the lung volumes will the respiratory therapist be most justified in using to guide interpretation of the test results? A) Vital capacity will equal the patient's combined inspiratory reserve, expiratory reserve, and tidal volume. B) Vital capacity will equal the total lung capacity. C) Resting tidal volume will exceed that of tidal volume during activity. D) Expiratory reserve will equal residual lung volume.

A) Vital capacity will equal the patient's combined inspiratory reserve, expiratory reserve, and tidal volume.

8. Laboratory testing is ordered for a male patient during a clinic visit for a routine follow-up assessment of hypertension. When interpreting lab values, the nurse knows that A) a normal value represents the test results that fall within the bell curve. B) if the lab result is above the 50% distribution, the result is considered elevated. C) all lab values are adjusted for gender and weight. D) if the result of a very sensitive test is negative, that does not mean the person is disease free.

A) a normal value represents the test results that fall within the bell curve.

15. A nurse practitioner is working in a crowded neighborhood where the population is primarily immigrants from China. The nurse has designed a research study to follow children from kindergarten to the age of 25. She is going to be looking at their diet, successful progression in school, health practices, and development of disease, to name a few items. This type of research is known as A) cohort study. B) cross-sectional study. C) case-control study. D) epidemiological study.

A) cohort study.

11. A male international business traveler has returned from a trip to Indonesia. While there, he hired a prostitute for companionship and engaged in unprotected sex on more than one occasion. Unbeknownst to him, this prostitute harbored the hepatitis C virus. Upon return to the United States, he exhibited no symptoms and returned to his usual activities. During this period of no outward symptoms, the man would be classified as being in A) the preclinical stage of disease. B) remission and unlikely to develop hepatitis C. C) the clinical disease stage of hepatitis C. D) the chronic phase of hepatitis C.

A) the preclinical stage of disease.

14. A 28-year-old man presents with complaints of diarrhea, fecal urgency, and weight loss. His stool is light colored and malodorous, and it tends to float and be difficult to flush. He has also noted tender, red bumps on his shins and complains of pain and stiffness in his elbows and knees. Sigmoidoscopy reveals discontinuous, granulomatous lesions; no blood is detected in his stool. Which of the following diagnoses would his care team first suspect? A) Crohn disease B) Ulcerative colitis C) Diverticulitis D) Colon cancer

Ans A Feedback: Crohn disease, like ulcerative colitis, causes diarrhea, fecal urgency, weight loss, and systemic symptoms such as erythema nodosum and arthritis. Unlike ulcerative colitis, it also causes steatorrhea but is not as likely to cause blood in the stool. The granulomatous "skip" lesions confirm the diagnosis of Crohn disease. Neither diverticulitis nor colon cancer would cause this combination of symptoms and signs.

3. A female neonate has been in respiratory distress since delivery and is unresponsive to oxygen therapy. Endoscopy has confirmed a diagnosis of esophageal atresia and tracheoesophageal fistulae (EA/TEF). Which of the following explanations should the care team provide to the infant's parents? A) "We will have to perform surgery to correct the hole in her throat to make sure that she is able to swallow and breathe normally." B) "This problem will require respiratory therapy and supplementary feeding, but it will likely resolve itself over time." C) "The biggest risk that your daughter will face until this is fixed is the danger of malnutrition and dehydration." D) "The priority in our immediate treatment prior to her surgery will be pain management, as the contents of her stomach can burn her lungs."

Ans A Feedback: EA/TFE necessitate surgery and preclude both normal respiration and swallowing. Aspiration is the primary immediate risk and the priority for treatment. Although the infant will require respiratory therapy to assist with proper tube placement and ventilator maintenance, the only effective treatment is surgery. Aspiration of feeding (aspiration pneumonia) is a major complication that can occur immediately and can be life threatening. Maintaining an open airway and adequate gas exchange are the priority nursing diagnoses for this infant.

7. A 68-year-old woman with a new onset of vascular dementia has recently begun retaining urine. Which of the following physiological phenomena would her care providers most realistically expect to currently occur as a result of her urinary retention? A) Hypertrophy of the bladder muscle and increased bladder wall thickness B) Decreased urine production and nitrogenous waste excretion by the kidneys C) Decompensation, bladder stretching, and high residual urine volume D) Overflow incontinence and loss of contraction power

Ans A Feedback: Early accompaniments to urinary retention include hypertrophy of the bladder muscle and increased thickness of the bladder wall. Renal effects are unlikely, and decompensation and loss of contraction power are most often later rather than early effects.

16. A number of clients on a geriatric subacute medical floor of a hospital have developed foul-smelling diarrhea over the last several days, and subsequent culture of stool samples has confirmed the presence of Clostridium difficile in each case. The care team in the unit would recognize that which of the following factors likely contributed to the health problem and would anticipate which of the following treatments? A) The use of broad-spectrum antibiotics likely played a role in the development of infections, and most clients would likely receive metronidazole as a treatment. B) Genetic predisposition and the presence of the bacterium in clients' normal flora likely contributed, and treatment would consist of broad-spectrum antibiotics. C) Poor hand washing practice on the part of care providers led to the outbreak, and treatment will consist of hydration and nutritional support. D) Ingestion of contaminated food probably contributed to the infections, and corticosteroids will be needed to treat them.

Ans A Feedback: Elimination of the normal intestinal flora by broad-spectrum antibiotics commonly precedes infection by C. difficile, and metronidazole is the normal treatment. Neither genetic predisposition nor ingestion of contaminated food is a likely factor. While poor hand hygiene can spread the bacteria, treatment for C. difficile necessitates antibiotics and not simply hydration and nutritional support. The treatment of C. difficile is with antibiotics. Metronidazole is the drug of choice with vancomycin, being reserved for people who cannot tolerate metronidazole. Corticosteroids will not kill the bacterium.

4. A stroke patient is having difficulty swallowing food and beverages. The patient complains that he feels like "the food is sticking to the back of his throat." Given this information, the priority nursing interventions would be to A) make the patient "nothing per os" (NPO) and call the physician. B) feed the patient while he is sitting in an upright position. C) add a thickening agent to all of the patient's beverages. D) warrant no action since this is a normal occurrence after a stroke.

Ans A Feedback: People with dysphagia usually complain of choking, coughing, or an abnormal sensation of food sticking in the back of the throat or upper chest when they swallow. A neuromuscular cause involves lesions of the CNS, such as a stroke, which often involve the cranial nerves that control swallowing. Feeding in upright position is good once it is determined by swallowing evaluation that the patient can swallow food without it going into the lungs. Likewise, thickening agents help dysphagia patients after a swallow evaluation has been performed. No action could put the patient at risk for aspiration pneumonia.

16. An 87-year-old male resident of an assisted living facility has been consistently continent of urine until the last several weeks. Which of the following actions by the care providers at the facility is the most likely priority? A) Performing a physical examination and history to determine the exact cause and character of the incontinence B) Providing client education focusing on the fact that occasional incontinence is a normal, age-related change C) Teaching the resident about protective pads, collection devices, and medications that may be effective D) Showing the resident the correct technique for exercises to improve bladder, sphincter, and pelvic floor tone

Ans A Feedback: The priority in the treatment of incontinence in the elderly is an acknowledgement that it is not an inevitability and that the exact causes should and most often can be identified. This identification by way of history-taking and examination would supersede teaching about protective devices or exercises.

9. A male patient has just been diagnosed with esophageal cancer. He knew that he was losing weight and fatigued most days, but he just attributed it to aging and working. The physician recommends chemotherapy and irradiation. However, the cancer has already metastasized. The patient asks the nurse what he can expect if he agrees to the treatments. The nurse responds, A) "The therapies may shrink the cancer." B) "The doctor is prescribing treatment measures to help you swallow better." C) "These therapies will most likely cure your cancer." D) "You need to talk with your physician some more. I will page him for you."

Ans A Feedback: The prognosis for people with cancer of the esophagus, although poor, has improved. Even with modern forms of therapy, the long-term survival is limited because, in many cases, the disease has already metastasized by the time the diagnosis is made. These therapies may help with food consumption, but that is not their primary purpose. Nor, will these therapies cure the cancer.

10. A 43-year-old male client has presented to the emergency department with vomiting that he claims is of a sudden onset. The client also states that the emesis has often contained frank blood in the hours prior to admission. His vital signs are stable with temperature 98.3°F, pulse 88, BP 140/87, and respiratory rate 18. Which of the following potential contributing factors would the health care team suspect first? A) Overuse of antacids B) Alcohol consumption C) Staphylococcal enterotoxins D) Effects of Helicobacter pylori

Ans B Feedback: Acute gastritis associated with alcohol use is characterized by intermittent vomiting and the possibility of hematemesis. Aspirin and H. pylori do not normally cause such an acute symptom onset, and infectious organisms do not normally cause bleeding of the stomach lining. A combination of calcium carbonate and magnesium is commonly found in antacids. Overdose of antacids can result in irregular heartbeat, poor balance, shallow, rapid breathing and stupor (lack of alertness).

19. The mother of a 19-week-old infant has brought her baby in for assessment to a pediatrician because of the baby's persistent weight loss and diarrhea. An intestinal biopsy has confirmed a diagnosis of celiac disease, and the child's mother is anxious to know what caused the disease. Which of the following aspects of the etiology of celiac disease would underlie the explanation that the physician provides? A) Bacterial or chemical invasion of the peritoneum leads to decreased nutrient absorption and transport. B) An inappropriate T-cell-mediated response results in increased levels of antibodies and an inflammatory response. C) Neurogenic or muscular inhibition of peristalsis results in inappropriate motility of ingested food in the lower small intestine and the colon. D) Inability to process or absorb the fat content of breast milk results in malnutrition and deficiency of fat-soluble vitamins.

Ans B Feedback: Celiac disease is rooted in an inappropriate immune response that initiates an inflammatory response, resulting in loss of absorptive villi. Bacterial or chemical invasion of the peritoneum is associated with peritonitis, while inhibition of peristalsis is associated with obstructions. An inability to process or absorb fat is associated with malabsorption syndrome.

2. During male ejaculation, which of the following statements addresses why sperm is not normally seen inside the bladder? A) The parasympathetic nervous system keeps the seminal fluid inside the urethra. B) The musculature of the trigone area, bladder neck, and prostatic urethra contract at the same time. C) With ejaculation, the male expels some urine along with the seminal fluid to wash any extra sperm out of the bladder. D) The detrusor muscle relaxes allowing for the closing of the sphincter at the base of the bladder.

Ans B Feedback: During male ejaculation, which is mediated by the SNS, the musculature of the trigone area and that of the bladder neck and prostatic urethra contracts and prevents the backflow of seminal fluid into the bladder.

11. A patient has recently been diagnosed with H. pylori gastritis. The nurse knows that this form of gastritis is usually treated with a combination of an antibiotic and A) antianxiety medications. B) proton pump inhibitors. C) lactulose, to reduce the blood ammonia levels. D) calcium carbonate, an antacid.

Ans B Feedback: H. pylori is associated with an increased risk of gastric adenocarcinoma, gastric atrophy, and peptic ulcer. It is less likely to contribute to IBD, esophagitis, or diverticular disease. Eradication of H. pylori is difficult. Treatment requires a combination therapy that includes the use of antibiotics and a proton pump inhibitor. The proton pump inhibitors have direct antimicrobial properties against H. pylori. Antianxiety medications will not kill the bacteria. H. pylori is not associated with elevated blood ammonia levels. Calcium carbonate is usually given to relieve heartburn caused by GERD.

2. A middle-aged male walks into the emergency department complaining of chest pain radiating to the neck, shortness of breath, and nausea. His heart rate is 120 and BP is 94/60. The ED physician recognizes the patient is having an acute MI with decreased cardiac output. The nurse identifies the nausea to be in response to A) the patient not having a very high pain tolerance. B) hypoxia exerting a direct effect on the chemoreceptor trigger zone. C) the patient not having digested his meal completely. D) fear of having to make major lifestyle changes.

Ans B Feedback: Hypoxia exerts a direct effect on the vomiting center (chemoreceptor trigger zone), producing nausea and vomiting. This direct effect probably accounts for the vomiting that occurs during periods of decreased cardiac output, shock, and environmental hypoxia. We are given no information about the patient's pain tolerance, when he last had a meal, or his routine lifestyle. This patient is going into shock (rapid pulse, low BP) that can result in shunting of blood away from the gut and other organs. During shock, the priority organs for oxygenation include the heart, brain, lungs, and kidneys.

13. Which of the following clients is most clearly displaying the signs and symptoms of irritable bowel disease (IBD)? A) A 32-year-old mother who complains of intermittent abdominal pain that is worse during her menstrual period B) A 51-year-old male who states that his stomach pain is in his lower abdomen, "comes and goes," and "feels more like a cramp than a dull ache" C) A 44-year-old man who works the evening shift at a factory and who states that his lower abdominal pain is much worse at night than during the day D) A 24-year-old man who has a stressful job but whose diarrhea and cramping do not worsen during periods of high stress

Ans B Feedback: IBD is commonly manifested as intermittent lower abdominal pain that feels like cramping. Defecation normally relieves the pain, and symptoms are normally not present at night or during sleep. Stress commonly exacerbates symptoms.

18. A 22-year-old student has developed a fever and diarrhea while on a backpacking trip in Southeast Asia. His oral temperature is 101.4°F. The diarrhea is bloody, frequent, and small in volume. These clinical manifestations are sufficiently distressing that he is visiting a local medical clinic in the area. Which of the following diagnoses best characterizes this health problem? A) Noninflammatory diarrhea B) Inflammatory diarrhea C) Factitious diarrhea D) Secretory diarrhea

Ans B Feedback: Inflammatory diarrhea is often characterized by small-volume diarrhea that is bloody and accompanied by a fever. Noninflammatory diarrhea is normally larger in volume and not bloody. Factitious diarrhea is normally attributable to laxative use, and secretory diarrhea is associated with increased secretory processes of the bowel; neither is likely to produce bloody stool.

8. A 51-year-old woman diagnosed with a cerebrovascular accident (CVA) 5 months prior is distressed that she has had several recent episodes of urinary incontinence. She has asked her nurse practitioner why this is the case. Which of the following statements best captures the fact that would underlie the nurse's response to the client? A) Neurological diseases like MS often result in flaccid bladder dysfunction. B) She may be unable to sense her bladder filling as a result of her MS. C) Lesions to the basal ganglia or extrapyramidal tract associated with MS inhibit detrusor contraction. D) Pathological reductions in bladder volume brought on my MS necessitate frequent micturition.

Ans B Feedback: MS may result in neurogenic bladder characterized by an inability to sense filling and consequent incontinence. She is not demonstrating the signs of a flaccid bladder, and lesions to the basal ganglia or extrapyramidal tract are associated with Parkinson disease, not MS. Her disease is unlikely to directly reduce bladder volume.

8. A 68-year-old African American man who has smoked for at least 50 years reports that lately he feels as though food is getting stuck in his throat. At first, this was a problem just with dry food, but now his morning oatmeal is getting "stuck." On questioning, he reports drinking at least three alcoholic beverages nearly every day. His problem is most likely A) achalasia. B) squamous cell carcinoma of the esophagus. C) dysphagia secondary to scleroderma. D) gastrointestinal reflux disease.

Ans B Feedback: Squamous cell carcinoma of the esophagus is the seventh leading cause of cancer death among men, particularly black men; mean age at diagnosis is 67 years. Alcohol and tobacco use are the main risk factors for this cancer, and dysphagia is a common presenting complaint. An esophageal motility disorder involves the smooth muscle layer of the esophagus and the lower esophageal splincter (LES). Achalasia is characterized by difficulty swallowing and regurgitation. GERD (gastroesophageal reflux disease) is a condition that causes the esophagus to become irritated and inflamed. Clients with GERD usually feel a burning in the chest or throat called heartburn. Sometimes, they taste stomach fluid in the back of the mouth.

14. A 55-year-old man has made an appointment to see his family physician because he has been awakening three to four times nightly to void and often has a sudden need to void with little warning during the day. What is the man's most likely diagnosis and possible underlying pathophysiological problem? A) Stress incontinence due to damage to CNS inhibitory pathways B) Overactive bladder that may result from both neurogenic and myogenic sources C) Overactive bladder due to intravesical pressure exceeding urethral pressure D) Overflow incontinence that can result from displacement of the angle between the bladder and the posterior proximal urethra

Ans B Feedback: The man's complaints are typical of overactive bladder, a condition that can result from the interaction of both the nervous control of bladder emptying and the muscles of the bladder itself. His symptoms are not characteristic of stress incontinence, and when intravesical pressure exceeds, urethral pressure overflow incontinence results. The angle between the bladder and the posterior proximal urethra is more commonly a factor in the continence of females.

4. An 82-year-old resident of a long-term care facility with a recent history of repeated urinary tract infections and restlessness is suspected of having urinary retention. Which of the following actions by the care team is most appropriate? A) Uroflowmetry to determine the rate of the client's urine flow B) Ultrasound bladder scanning to determine the residual volume of urine after voiding C) Renal ultrasound aimed at identifying acute or chronic kidney disease D) Urinalysis focusing on the presence of or absence of microorganisms, blood, or white cells in the man's urine

Ans B Feedback: Ultrasound bladder scanning yields a fast and noninvasive indication of whether or not an individual is adequately emptying his or her bladder with each void. Uroflowmetry would be less indicative of whether the man is retaining, and renal ultrasound would address deficits in urine production rather than bladder emptying. Urinalysis would be useful in the diagnosis of infections and/or renal issues more than deficiencies in bladder emptying.

15. Because they strengthen the pelvic floor muscles, Kegel exercises are most likely to help A) overflow incontinence. B) urge incontinence. C) stress incontinence. D) mixed incontinence.

Ans C Feedback: : Stress incontinence is commonly caused by weak pelvic floor muscles, which allow the angle between the bladder and the posterior proximal urethra to change so that the bladder and urethra are positioned for voiding when some activity increases intra-abdominal pressure. Overflow incontinence results when the bladder becomes distended and detrusor activity is absent. Urge incontinence is probably related to CNS control of bladder sensation and emptying or to the smooth muscle of the bladder. Mixed incontinence, a combination of stress and urge incontinence, probably has more than one cause.

5. A nurse practitioner is providing care for a male client with a long-standing hiatal hernia. Which of the following statements most accurately captures an aspect of the pathophysiology of hiatal hernias? A) Paraesophageal hiatal hernias are common and are normally not treated if the client is asymptomatic. B) The root causes of hiatal hernias are normally treatable with medication. C) If esophageal acid clearance is impaired, esophagitis can result. D) An incompetent pyloric sphincter and high-fat diet are commonly implicated in the development of hiatal hernias.

Ans C Feedback: Erosive esophagitis can be a complication of hiatal hernias if esophageal acid clearance is significantly impaired. Paraesophageal hiatal hernias are more serious than the sliding variety and require treatment. The root cause of hiatal hernias, herniation of the stomach through the diaphragm, is not normally amenable to treatment with medication. The pyloric sphincter is not associated with hiatal hernias.

7. Parents of a 20-month-old infant report that he refuses food or eats poorly and that he grimaces when he swallows. He also is irritable and cries a lot. The mother is worried that he ate something inappropriate this morning, because he vomited something that looked like coffee grounds. Which of the following health problems would the care team first suspect? A) Rotavirus infection B) Appendicitis C) Esophagitis from gastrointestinal reflux D) Hirschsprung disease

Ans C Feedback: Esophagitis secondary to reflux can cause feeding problems, early satiety, and hematemesis. Infants may demonstrate signs of pain when swallowing and may be irritable and cry frequently. Rotavirus causes diarrhea and vomiting, but not the other symptoms. Appendicitis is inflammation of the appendix. Appendicitis usually starts with the main symptom of pain around the navel that moves to the lower right abdomen. Hirschsprung disease is a blockage of the large intestine due to improper muscle movement in the bowel. It is a congenital condition, which means it is present from birth. In Hirschsprung disease, the nerves are missing from a part of the bowel. One primary s/s is a failure to pass meconium shortly after birth.

12. Following a history of gastric pain and an endoscopy, a client has been diagnosed with a duodenal peptic ulcer. Which of the following teaching points should his caregiver provide? A) "While your diet most certainly contributed to this problem, the good news is that changing your diet can help solve it." B) "Ulcers like yours do not penetrate all layers of the stomach or duodenum, so you don't have to worry about losing too much blood." C) "Your family history, your smoking history, and NSAID use may all have contributed to this problem." D) "While there aren't really any effective medications for these ulcers, changes in lifestyle can keep them well controlled."

Ans C Feedback: Family history, NSAID use, and smoking have all been identified as contributing factors in the development of peptic ulcers. Diet therapy has not been shown to be effective, and duodenal peptic ulcers are more common than the gastric variant. Perforation occurs when an ulcer erodes through all layers of the stomach or duodenum wall. When perforation occurs in older adults, their mortality is significantly increased. Effective medication regimens are available with antacids, H2-receptor antagonists or proton pump inhibitors being the most common medications used.

12. A 24-year-old man is currently in a rehabilitation facility following a spinal cord injury at level T2. He is discussing his long-term options for continence management. Which of the following statements by the client demonstrates he has a clear understanding of the issue? A) "Self-catheterization can limit the recovery of my neural pathways that control my voiding if I do it too often." B) "It's critical that intermittent catheterization be performed using sterile technique." C) "An indwelling catheter certainly would work well, but it comes with a number of risks and possible complications." D) "An indwelling urethral catheter is the option that best minimizes my chance of a urinary tract infection."

Ans C Feedback: Indwelling catheters carry a risk of infections and kidney stones. Catheterization does not influence the activity of the neural pathways, and intermittent catheterization can be performed using clean technique. Indwelling urethral catheters carry a high risk of urinary tract infections.

3. A 61-year-old woman who has had an upper respiratory infection for several weeks has presented to her family physician with complaints of a recent onset of urinary retention. She reveals to her physician that she has been taking nonprescription cold medications over and above the suggested dose for the past 2 weeks. Which of the following phenomena will her physician most likely suspect is contributing to her urinary retention? A) Cholinergic actions of the cold medicine are triggering internal and external sphincter contraction. B) Antihistamine effects inhibit communication between the pons and the thoracolumbar cord. C) The anticholinergic effects of the medication are impairing normal bladder function. D) Over-the-counter medications such as cold medicine stimulate the parasympathetic nervous system and inhibit bladder emptying.

Ans C Feedback: Many over-the-counter cold medications have an anticholinergic effect that interferes with normal bladder emptying. These effects on micturition are not a result of cholinergic actions or miscommunication between the pontine micturition center and the spinal cord. Stimulation of the parasympathetic nervous system would tend to increase rather than decrease bladder emptying.

19. A 63-year-old woman has visited a physician because she has been intermittently passing blood-tinged urine over the last several weeks, and cytology has confirmed a diagnosis of invasive bladder cancer. Which of the following statements by the physician is most accurate? A) "There are new and highly effective chemotherapy regimens that we will investigate." B) "Fortunately, bladder cancer has a very low mortality rate, and successful treatment is nearly always possible." C) "It's likely that you'll need surgery, possibly a procedure called a cystectomy." D) "Unfortunately, there are nearly no treatment options for this type of cancer, but we will focus on addressing your symptoms."

Ans C Feedback: Surgical interventions are common in the treatment of bladder cancer. Effective chemotherapeutic regimens are not yet available, though there are certainly treatment options. The mortality rate of bladder cancer is high, at around 25%.

1. When explaining about the passage of urine to a group of nursing students, the clinic nurse asks them which muscle is primarily responsible for micturition? Their correct reply is the A) urinary vesicle. B) trigone. C) detrusor. D) external sphincter.

Ans C Feedback: The detrusor muscle is the muscle of micturition. When it contracts, urine is expelled from the bladder. External sphincter is a circular muscle that surrounds the urethra distal to the base of the bladder and can stop micturition when it is occurring. Trigone is a smooth triangular area that is bounded by the ureters and the urethra. Urinary vesicle is another name for the bladder.

20. A 71-year-old male has been recently diagnosed with a stage III tumor of colorectal cancer and is attempting to increase his knowledge base of his diagnosis. Which of the following statements about colorectal cancer demonstrates a sound understanding of the disease? A) "If accurate screening test for this type of cancer existed, it could likely have been caught earlier." B) "The NSAIDs and aspirin that I've been taking for many years probably contributing to me getting cancer." C) "While diet is thought to play a role in the development of colorectal cancer, the ultimate causes are largely unknown." D) "A large majority of patients who have my type of colon cancer survive to live many more years."

Ans C Feedback: The etiology of cancer of the colon and rectum remains largely unidentified, though dietary factors are thought to exist. The prognosis, especially with stage III tumors, is poor. Simple and accurate screening tests do exist for colorectal cancer, while drugs are not implicated in the etiology.

9. A patient who has suffered a spinal cord injury at C4 is experiencing a sudden change in condition. His BP is 186/101; heart rate is 45; and he is profusely sweating and complaining of "not feeling right." The nurse should A) call a "Code Blue." B) page physician stat. and ask for an antihypertensive medication. C) palpate his bladder for overdistention. D) place his bed flat and elevate the foot of the bed.

Ans C Feedback: The most common causes of spastic bladder dysfunction are spinal cord lesions such as spinal cord injury, herniated intervertebral disk, vascular lesions, tumors, and myelitis. Because the injury interrupts CNS control of sympathetic reflexes in the spinal cord, severe hypertension, bradycardia, and sweating can be triggered by insertion of a catheter or mild overdistention of the bladder. The patient does not qualify for a Code Blue since he still has a pulse and is breathing. Antihypertensive medication is not necessary if the bladder is emptied. Placing him flat with the foot of the bed elevated will not help this situation.

18. When teaching a community education class about the seven warning signs of cancer, the nurse will note that the most common sign of bladder cancer is A) inability to empty the bladder fully. B) colic spasms of the ureters. C) painless bloody urine. D) passage of large clots after voiding.

Ans C Feedback: The most common sign of bladder cancer is painless hematuria. Gross hematuria is a presenting sign in the majority of people with the disease, and microscopic hematuria is present in most others. Answer choice A refers to flaccid bladder; answer choice B refers to kidney stones; answer choice D refers to clots that are usually seen after surgery such as TURP where bladder irrigation is called for to prevent the clots from blocking urine output.

1. The nurse walks into a room and finds the patient forcefully expelling stomach contents into a wash basin. When documenting this occurrence, the nurse will use the term A) nauseous. B) retching. C) vomiting. D) expatriate.

Ans C Feedback: Vomiting or emesis is the sudden and forceful oral expulsion of the contents of the stomach. It is usually preceded by nausea. Nausea is a subjective and unpleasant sensation. Retching consists of rhythmic spasmodic movements of the diaphragm, chest wall, and abdominal muscles. It usually precedes or alternates with periods of vomiting. Expatriate means to banish or withdraw.

17. A patient asks the nurse what it means when the doctor said that he had adenocarcinoma of the bladder. Reviewing the pathophysiologic principles behind this type of cancer, the nurse knows A) it is a low-grade tumor that is readily cured with bladder surgery. B) after resection of the cancer, the prognosis is excellent with this type of cancer cell. C) that these types of cancer cells are very invasive to the tissue; therefore, the entire bladder must be removed. D) this is a rare but highly metastatic tumor that has a very poor prognosis.

Ans D Feedback: Adenocarcinoma is rare and highly metastatic. Answer choices A and B relate to urothelial carcinoma; answer choice C relates to squamous cell carcinoma.

10. A middle-aged man with diabetes reports that he must strain to urinate and that his urine stream is weak and dribbling. He also reports feeling that his bladder never really empties. The nurse knows that all of his complaints are likely caused by which of the following medical diagnoses? A) Detrusor muscle areflexia B) Detrusor-sphincter dyssynergia C) Uninhibited neurogenic bladder D) Bladder atony with dysfunction

Ans D Feedback: Diabetes causes peripheral neuropathy, which can affect the sensory axons of the urinary bladder. Bladder atony with dysfunction is a frequent complication of diabetes mellitus.

11. A diabetes education nurse is teaching a group of recently diagnosed diabetics about the potential genitourinary complications of diabetes and the consequent importance of vigilant blood glucose control. Which of the following teaching points best conveys an aspect of bladder dysfunction and diabetes mellitus? A) "People with diabetes are highly susceptible to urethral obstructions, and these can heal more slowly and cause more damage than in people without diabetes." B) "High blood sugar results in a high glucose level in your urine, and this can make your bladder muscle less able to fully empty the bladder." C) "Many people with diabetes find it necessary to live with an indwelling catheter to ensure their bladders do not become too full." D) "It's important for you to empty your bladder frequently because diabetes carries risks of kidney damage that can be exacerbated by incomplete bladder emptying."

Ans D Feedback: Diabetics are vulnerable to peripheral neuropathies that can be somewhat mitigated by regular voiding; they are also especially vulnerable to renal damage from high blood sugars, a situation that is worsened when accompanied by incomplete bladder emptying. Urethral obstructions are not a noted complication of diabetes, and indwelling catheter placement is not normally necessary. High blood sugars do not necessarily yield high-glucose urine, and the bladder deficits associated with diabetes are neurological in nature rather than a result of particular urine chemistry.

6. After several months of persistent heartburn, a 57-year-old female client has been diagnosed with gastroesophageal reflux disease (GERD). Which of the following treatment regimens is likely to best address the woman's health problem? A) Surgical correction of the incompetent pylorus B) Antacids; avoiding positions that exacerbate reflux; a soft-textured diet C) Weight loss and administration of calcium channel blocking medications D) Proton pump inhibitors; avoiding large meals; remaining upright after meals

Ans D Feedback: Proton pump inhibitors block the final stage of gastric acid production, effectively controlling the root cause of the esophageal damage associated with GERD. The pylorus is not involved, and a soft diet is not indicated. Calcium channel-blocking drugs would not address the problem. Calcium channel blockers are primarily heart disease drugs that relax blood vessels and increase the supply of blood and oxygen to the heart while also reducing the heart's workload.

5. When explaining a cystometry test to measure bladder pressure during filling and voiding in a normal adult, the nurse informs the nursing students that the normal capacity when adults have a desire to void is A) 100 to 150 mL. B) 200 to 250 mL. C) 300 to 399 mL. D) 400 to 500 mL.

Ans D Feedback: The desire to void occurs when the bladder is full (normal capacity is approximately 400 to 500 mL). At this point, a definite sensation of fullness occurs; the pressure rises sharply to 40 to 100 cm H2O; and voiding occurs around the catheter.

15. A 20-year-old woman has visited her family physician due to occasional bouts of bloody diarrhea over the past several weeks, a phenomenon that she experienced 2 years prior as well. Her physician has diagnosed her with ulcerative colitis based on her history and visualization of the affected region by colonoscopy and sigmoidoscopy. Which of the following pathophysiological phenomena is most likely to underlie the client's health problem? A) Fissures and crevices developing in the mucosa that are seen as a characteristic "cobblestone" appearance B) Erosion of the endothelial lining of the distal small intestine by a combination of genetic, autoimmune, and environmental factors C) Compromise of the mucosal layer of the large intestinal surface by the effects of H. pylori D) Mucosal hemorrhages that have developed into crypt abscesses, which have in turn necrotized and ulcerated

Ans D Feedback: The etiology and course of ulcerative colitis involves mucosal hemorrhages developing into crypt abscesses, with consequent necrosis and ulceration. "Cobblestone" appearance of intestinal mucosa is associated with Crohn disease. Ulcerative colitis is confined to the colon and rectum, and H. pylori is not commonly implicated in the etiology.

1. As part of presurgical teaching for patients who are about to undergo a coronary artery bypass graft, a nurse is performing an education session with a group of surgical candidates. Which of the following teaching points best conveys an aspect of the human circulatory system? A) "The blood pressure varies widely between arteries and veins and between pulmonary and systemic circulation." B) "Only around one quarter of your blood is in your heart at any given time." C) "Blood pressure and blood volume roughly mimic one another at any given location in the circulatory system." D) "Left-sided and right-sided pumping action on each beat of the heart must equal each other to ensure adequate blood distribution."

Ans: A

10. A hospital client with a diagnosis of chronic renal failure has orders for measurement of her serum electrolyte levels three times per week. Which of the following statements best captures the relationship between renal failure and sodium regulation? A) Clients with advanced renal failure are prone to hyponatremia because of impaired tubular reabsorption. B) Renal clients often require a sodium-restricted diet to minimize the excretion load on remaining nephrons. C) Clients with renal failure often maintain high sodium levels because of decreased excretion. D) Restricting sodium intake helps to preserve nephron function and has the additional benefit of lowering blood pressure.

Ans: A

10. During a routine physical exam for a patient diagnosed with hypertension, the nurse practitioner will be most concerned if which of the following assessments are found? A) Noted hemorrhages and microaneurysms during evaluation of the internal eye B) Unable to feel vibrations when a tuning fork is placed on the skull C) Inability to locate the kidneys with deep palpation to the abdomen D) Slight increase in the number of varicose veins noted bilaterally

Ans: A

10. Emergency medical technicians respond to a call to find an 80-year-old man who is showing signs and symptoms of severe shock. Which of the following phenomena is most likely taking place? A) The man's - and -adrenergic receptors have been activated, resulting in vasoconstriction and increased heart rate. B) Hemolysis and blood pooling are taking place in the man's peripheral circulation. C) Bronchoconstriction and hyperventilation are initiated as a compensatory mechanism. D) Intracellular potassium and extracellular sodium levels are rising as a result of sodium-potassium pump failure.

Ans: A

12. Which of the following hypertensive individuals is most likely to have his or her high blood pressure diagnosed as secondary rather than essential? A) A 51-year-old male who has been diagnosed with glomerulonephritis B) An African American man who leads a sedentary lifestyle C) A 69-year-old woman with a diagnosis of cardiometabolic syndrome D) A 40-year-old smoker who eats excessive amounts of salt and saturated fats

Ans: A

13. A 31-year-old African American female who is in her 30th week of pregnancy has been diagnosed with peripartum cardiomyopathy. Which of the following statements best captures an aspect of peripartum cardiomyopathy? A) Her diagnosis might be attributable to a disordered immune response, nutritional factors, or infectious processes. B) Treatment is possible in postpartum women, but antepartum women are dependent on spontaneous resolution of the problem. C) Mortality exceeds 50%, and very few surviving women regain normal heart function. D) Symptomatology mimics that of stable angina and is diagnosed and treated similarly.

Ans: A

13. A patient arrives at the ED complaining of numbness in the left lower leg. Upon assessment, the nurse finds the lower left leg to be cold to touch, pedal and posterior tibial pulses nonpalpable, and a sharp line of paralysis/paresthesia. The nurse's next action is based on the fact that A) acute arterial occlusion is a medical emergency requiring immediate intervention to restore blood flow. B) submersion in a whirlpool with warm water will improve the venous blood flow and restore pulses. C) the immediate infusion of tissue plasminogen activator (tPA) will not correct the problem and should only be used for CVAs. D) administration of an aspirin and sublingual nitroglycerin will vasodilate the artery to restore perfusion.

Ans: A

13. A patient with small cell lung cancer (SCLC) has developed a paraneoplastic syndrome called Cushing syndrome. Based on this new complication, the nurse will likely assess which of the following clinical manifestations of Cushing syndrome? A) Weight gain, moon face, buffalo hump, and purple striae on the abdomen B) Bilateral edema in the arms, swollen face, and protruding eyes C) Severe bone/joint pain, nausea/vomiting, and polyuria D) Tetany, new-onset seizure activity, emotional lability, and extrapyramidal symptoms

Ans: A

14. All of the following interventions are ordered stat. for a patient stung by a bee who is experiencing severe respiratory distress and faintness. Which priority intervention will the nurse administer first? A) Epinephrine (Adrenalin) B) Normal saline infusion C) Dexamethasone (Decadron) D) Diphenhydramine (Benadryl)

Ans: A

14. An IV drug abuser walks into the ED telling the nurse that, "he is sick." He looks feverish with flushed, moist skin; dehydrated with dry lips/mucous membranes; and fatigued. The assessment reveals a loud murmur. An echocardiogram was ordered that shows a large vegetation growing on his mitral valve. The patient is admitted to the ICU. The nurse will be assessing this patient for which possible life-threatening complications? A) Systemic emboli, especially to the brain B) Petechial hemorrhages under the skin and nail beds C) GI upset from the massive amount of antibiotics required to kill the bacteria D) Pancreas enlargement due to increased need for insulin secretion

Ans: A

16. A 4-year-old boy who has been deaf since birth and has bilateral cataracts has been brought to the emergency department by his mother because she noticed blood in the toilet after he last voided. Urinalysis confirms heavy microscopic hematuria as well as proteinuria. What will the health care team's initial differential diagnosis most likely be? A) Alport syndrome B) Systemic lupus erythematosus glomerulonephritis C) Henoch-Schönlein purpura nephritis D) Immunoglobulin A nephropathy

Ans: A

16. A nurse who provides care in a geriatric subacute medicine unit of a hospital has noted that a large number of patients receive -adrenergic blocking medications such as metoprolol. Which of the following statements best conveys an aspect of the use of beta-blockers? A) They can be used to treat supraventricular arrhythmias and decrease automaticity by depressing phase 4 of the action potential. B) They inhibit the potassium current and repolarization, extending the action potential and refractoriness. C) They counteract arrhythmias and tachycardias by increasing vagal stimulation. D) They decrease myocardial oxygen demand by blocking the release of intracellular calcium ions.

Ans: A

16. Which of the following phenomena contributes to the difficulties with absorption, distribution, and elimination of drugs that are associated with kidney disease? A) Reductions in plasma proteins increase the amount of free drug and decrease the amount of protein-bound drug. B) Acute tubular necrosis is associated with impaired drug reabsorption through the tubular epithelium. C) Decreased retention by the kidneys often renders normal drug dosages ineffective. D) Dialysis removes active metabolites from circulation minimizing therapeutic effect.

Ans: A

17. A patient, who is experiencing some angina associated with atrial tachycardia, has been placed on verapamil (Calan), a calcium channel blocker. Knowing that this medication blocks the slow calcium channels, thereby depressing phase 4 and lengthening phases 1 and 2 action potential, the nurse should assess this patient for which of the following adverse reactions? A) Bradycardia B) Ventricular tachycardia C) Sudden cardiac death D) Increased cardiac output

Ans: A

17. Which of the following clients in a hospital medical unit is most clearly demonstrating the signs and symptoms of liver failure? An adult with A) low hemoglobin levels, low platelet levels, and spider angiomas present. B) blood pressure of 189/103, jaundice, and multiple thromboses. C) sudden onset of confusion, a history of alcohol abuse, and low levels of serum AST and ALT. D) ascites, fever, and recent onset of atrial fibrillation.

Ans: A

17. Which of the following individuals is most likely to be experiencing vasodilation? A) A 51-year-old man with a history of hypertension who is taking a medication that blocks the effect of the renin-angiotensin-aldosterone system B) A 9-year-old boy who has been given an injection of epinephrine to preclude an anaphylactic reaction to a bee sting C) A 30-year-old woman who takes antihistamines to treat her seasonal allergies D) A 32-year-old man who takes a selective serotonin reuptake inhibitor for the treatment of depression

Ans: A

18. A 44-year-old female who is on her feet for the duration of her entire work week has developed varicose veins in her legs. What teaching point would her care provider be most justified in emphasizing to the woman? A) "Once you have varicose veins, there's little that can be done to reverse them." B) "Your varicose veins are likely a consequence of an existing cardiac problem." C) "If you're able to stay off your feet and wear tight stockings, normal vein tone can be reestablished." D) "The use of blood thinner medications will likely relieve the backflow that is causing your varicose veins."

Ans: A

18. A pediatrician is teaching a group of medical students about some of the particularities of heart failure in children as compared with older adults. Which of the physician's following statements best captures an aspect of these differences? A) "You'll find that in pediatric patients, pulmonary edema is more often interstitial rather than alveolar, so you often won't hear crackles." B) "Because of their higher relative blood volume, jugular venous distention is a better assessment technique for suspected heart failure in young patients." C) "Signs and symptoms in children may sometimes mimic those of shock, with a low blood pressure and high heart rate." D) "Fever is a sign of heart failure in children that you are unlikely to see in older adults."

Ans: A

18. A premature infant on mechanical ventilation has developed bronchopulmonary dysplasia (BPD) and is showing signs and symptoms of hypoxemia, low lung compliance, and respiratory distress. Which of the following is the most likely contributor to the infant's present health problem? A) High-inspired oxygen concentration and injury from positive-pressure ventilation B) Failure to administer corticosteroids to the infant in utero C) Insufficient surfactant production and insufficient surfactant therapy D) Insufficient supplemental oxygen therapy

Ans: A

19. A 1-year-old baby boy with renal dysplasia risks end-stage renal disease unless intervention occurs. Which of the following treatment options is his care team most likely to reject? A) Dietary restriction plus erythropoietin B) Continuous cyclic peritoneal dialysis C) Renal transplantation D) Continuous ambulatory peritoneal dialysis

Ans: A

2. Which of the following patients who presented to a walk-in medical clinic is most likely to be diagnosed with a rhinosinusitis rather than a common cold? A) A man complaining of general fatigue, a headache, and facial pain with a temperature of 100.9°F B) A woman presenting with malaise, lethargy, and copious nasal secretions C) A man with a dry, stuffy nasopharynx, a sore throat, and temperature of 98.9°F D) A woman complaining of generalized aches and who has a hoarse voice and reddened, painful upper airways

Ans: A

2. Which of the following statements describes phase 4 of the action potential of cells in the sinoatrial (SA) node? A) A slow depolarization occurs when Na+ is transported out of the cell and K+ moves back in, resulting in resting membrane potential. B) The cells are capable of responding to a greater than normal stimulus before the resting membrane potential is reached. C) The fast sodium channels in the cellular membranes close, causing an abrupt decrease in intracellular positivity. D) Potassium permeability is allowing the cell membrane to remain depolarized, and Ca2+ channel opens moving Ca2+ back into the cell.

Ans: A

20. A physician is explaining to a group of medical students the concept of Virchow triad as it applies to venous thrombosis. Which of the following clinical observations of a 50-year-old male client is most likely unrelated to a component of Virchow triad? A) The man has decreased cardiac output and an ejection fraction of 30%. B) The man's prothrombin time and international normalized ratio (INR) are both low. C) The man has a previous history of a dissecting aneurysm. D) There is bilateral, brown pigmentation of his lower legs.

Ans: A

20. Knowing the high incidence and prevalence of heart failure among the elderly, the manager of a long-term care home has organized a workshop on the identification of early signs and symptoms of heart failure. Which of the following teaching points is most accurate? A) "Displays of aggression, confusion, and restlessness when the resident has no history of such behavior can be a sign of heart failure." B) "Heart failure will often first show up with persistent coughing and lung crackles." C) "Residents in early heart failure will often be flushed and have warm skin and a fever." D) "Complaints of chest pain are actually more often related to heart failure than to myocardial infarction."

Ans: A

3. A nurse is providing care for a patient who has been admitted with a newly diagnosed bilateral pleural effusion. Which of the following findings from the nurse's initial assessment of the patient is incongruent with the patient's diagnosis and would require further investigation? A) The client complains of sharp pain exacerbated by deep inspiration. B) The client's breath sounds are diminished on auscultation. C) Pulse oximetry indicates that the client is hypoxemic. D) The client complains of dyspnea and increased work of breathing.

Ans: A

3. A patient being seen in the clinic has just received his routine lab results. The patient has been told that his cholesterol level is extremely elevated. The physician plans to give the patient a prescription for medication to help control this condition. Which of the following medications should the nurse prepare to educate the patient on? A) Atorvastatin (Lipitor) B) Abilify (aripiprazole) C) Pancrecarb (Pancrelipase) D) Pancrelipase (Ultrase)

Ans: A

3. The nurse practitioner working in an overnight sleep lab is assessing and diagnosing patients with sleep apnea. During this diagnostic procedure, the nurse notes that a patient's blood pressure is 162/97. The nurse explains this connection to the patient based on which of the following pathophysiological principles? A) During apneic periods, the patient experiences hypoxemia that stimulates chemoreceptors to induce vasoconstriction. B) When the patient starts to snore, his epiglottis is closed over the trachea. C) When the airway is obstructed, specialized cells located in the back of the throat send signals to the kidney to increase pulse rate. D) When airways are obstructed, the body will retain extracellular fluid so that this fluid can be shifted to intravascular space to increase volume.

Ans: A

3. Which of the following statements most accurately captures a principle of blood flow? A) With constant pressure, a small increase in vessel radius results in an exponential increase in blood flow. B) Blood flow is primarily determined by blood viscosity and temperature. C) Blood flows most quickly in the small diameter peripheral capillaries. D) Smaller cross-sectional vessel area is associated with lower flow velocity.

Ans: A

4. A 72-year-old female has been told by her physician that she has a new heart murmur that requires her to go visit a cardiologist. Upon examination, the cardiologist informs the patient that she has aortic stenosis. After the cardiologist has left the room, the patient asks, "What caused this [aortic stenosis] to happen now?" The clinic nurse responds, A) "Heart murmurs result from tumultuous flow through a diseased heart valve that is too narrow and stiff. This flow causes a vibration called a murmur." B) "Aortic stenosis is commonly seen in elderly patients. Basically, there is a blockage in the valve that is causing blood to pool, causing decreased velocity of flow." C) "This is caused by a tear in one of the papillary muscles attached to the valve. They can do a procedure where they thread a catheter into the heart and reattach the muscle ends." D) "Because of the high amount of energy it takes to push blood through the aortic valve to the body, your valve is just had to work too hard and it is weakening."

Ans: A

4. A family physician is performing patient teaching about the influenza virus with each patient who has come to the clinic to receive that year's vaccine. Which of the following statements by the patient best reflects an accurate understanding of the flu virus? A) "I could come down with viral or bacterial pneumonia as a result of a bad flu bug." B) "I know my vaccination is especially important since there aren't any drugs that can treat the flu once I get sick with it." C) "The emphasis on bundling up, staying warm, and drinking lots of fluids is outdated and actually ineffective." D) "Like all vaccines, it is ideal if everyone in a population gets immunized against the flu."

Ans: A

4. As part of the diagnostic workup for a male client with a complex history of cardiovascular disease, the care team has identified the need for a record of the electrical activity of his heart, insight into the metabolism of his myocardium, and physical measurements and imaging of his heart. Which of the following series of tests is most likely to provide the needed data for his diagnosis and care? A) Echocardiogram, PET scan, ECG B) Ambulatory ECG, cardiac MRI, echocardiogram C) Serum creatinine levels, chest auscultation, myocardial perfusion scintigraphy D) Cardiac catheterization, cardiac CT, exercise stress testing

Ans: A

5. A 68-year-old male complains to his family physician that when he tests his blood pressure using a machine at his pharmacy, his heart rate is nearly always very low. At other times, he feels that his heart is racing, and it also seems to pause at times. The man has also had occasionally light-headedness and a recent syncopal episode. What is this client's most likely diagnosis and the phenomenon underlying it? A) Sick sinus syndrome as a result of a disease of his sinus node and atrial or junctional arrhythmias B) Ventricular arrhythmia as a result of alternating vagal and sympathetic stimulation C) Torsade de pointes as a result of disease of the bundle of His D) Premature atrial contractions that vacillate between tachycardic and bradycardic episodes as a consequence of an infectious process

Ans: A

5. A 73-year-old man presents to his family physician with complaints of recent urinary hesitation and is eventually diagnosed with benign prostatic hyperplasia (BPH). Which of the following clinical consequences would his care provider expect prior to the resolution of his health problem? A) Hydroureter and pain B) Development of renal calculi and renal cysts C) Unilateral hydronephrosis and pain D) Development of glomerulonephritis or nephrotic syndrome

Ans: A

6. A 6-year-old boy has been brought to the emergency department by ambulance after his mother discovered that his heart rate was "so fast I couldn't even count it." The child was determined to be in atrial flutter, and his mother is seeking an explanation from the health care team. Which of the following points should underlie an explanation to the mother? A) The child is experiencing a reentry rhythm in his right atrium. B) The resolution of the problem is dependent on spontaneous recovery and is resistant to pacing interventions. C) The child is likely to have a normal ECG apart from the rapid heart rate. D) The boy's atria are experiencing abnormal sympathetic stimulation.

Ans: A

6. A nurse educator in a geriatric medicine unit of a hospital is teaching a group of new graduates specific assessment criteria related to heart failure. Which of the following assessment criteria should the nurses prioritize in their practice? A) Measurement of urine output and mental status assessment B) Pupil response and counting the patient's apical heart rate C) Palpation of pedal (foot) pulses and pain assessment D) Activity tolerance and integumentary inspection

Ans: A

6. Following the diagnosis of acute renal failure, the nurse knows that one of the earliest manifestations of residual tubular damage is which of the following lab/diagnostic results? A) Elevated blood urea nitrogen (BUN) B) Serum creatinine elevation C) Inability to concentrate urine D) Reduced glomerular filtration rate

Ans: A

7. A 29-year-old female has been admitted to the emergency department following a suicide attempt by overdose of acetaminophen. What changes in the client's liver and diagnostic results would the care team most likely anticipate? A) Hepatocellular necrosis evidenced by increased ALT and AST levels B) Allergic inflammation accompanied by an increase in serum IgE and basophils C) Cholestatic reaction with increased bilirubin count D) Rapid onset of hepatitis and increased GGT, ALT, and bilirubin

Ans: A

7. Which of the following medications will likely be prescribed for a patient with elevated LDL and triglyceride levels? A) Zocor (simvastatin), an HMG-CoA reductase inhibitor or "statin" B) Cholestyramine (Questran), a bile acid sequestrant C) Nicotinic acid (Niacin), a B vitamin D) Fenofibrate (Tricor), a fibric acid

Ans: A

8. A female older adult client has presented with a new onset of shortness of breath, and her physician has ordered measurement of her brain natriuretic peptide (BNP) levels along with other diagnostic tests. What is the most accurate rationale for the physician's choice of blood work? A) BNP is released as a compensatory mechanism during heart failure, and measuring it can help differentiate the client's dyspnea from a respiratory pathology. B) BNP is an indirect indicator of the effectiveness of the renin-angiotensin-aldosterone (RAA) system in compensating for heart failure. C) BNP levels correlate with the client's risk of developing cognitive deficits secondary to heart failure and consequent brain hypoxia. D) BNP becomes elevated in cases of cardiac asthma, Cheyne-Stokes respirations, and acute pulmonary edema, and measurement can gauge the severity of pulmonary effects.

Ans: A

8. A physician is providing care for several patients on a medical unit of a hospital. In which of the following patient situations would the physician most likelyrule out hypertension as a contributing factor? A) A 61-year-old man who has a heart valve infection and recurrent fever B) An 81-year-old woman who has had an ischemic stroke and has consequent one-sided weakness C) A 44-year-old man awaiting a kidney transplant who requires hemodialysis three times per week D) A 66-year-old woman with poorly controlled angina and consequent limited activity tolerance

Ans: A

8. The mother of a 7-year-old boy who has recently been diagnosed with childhood asthma has come to the education center to learn more about her son's condition. Which of the following teaching points is most justifiable? A) "Research has shown that viruses may actually be a factor in many children's asthma." B) "The most reliable indicator that your child is having an asthma attack is audible wheezing." C) "Steroids that your child can inhale will likely be the first line of defense." D) "Your son will likely need to limit or avoid exercise and sports."

Ans: A

9. A patient is reading a brochure on atherosclerosis while in the waiting room of medical clinic. Which of the following excerpts from the educational brochure warrants correction? A) "Because smoking causes a permanent increase in your risk of heart disease, it's best not to start." B) "All things being equal, men have a higher risk of coronary heart disease than perimenopausal women." C) "High blood pressure often accompanies, or even causes, clogging of the arteries." D) "Every bit that you can lower your cholesterol means that you'll have a lower risk of developing heart disease."

Ans: A

15. Chronic anxiety and stress contribute to ulcers. Which of the following effects of the sympathetic nervous system is most responsible for this effect? A) Inhibition of the actions of Brunner glands B) Overstimulation of the oxyntic glands C) Suppression of cholecystokinin D) Inflammation of the parotid glands

Ans: A Feedback: Brunner glands, which produce large amounts of alkaline mucus that protects the duodenum from acid and digestive enzymes, are strongly affected by sympathetic stimulation, which causes a marked decrease in mucus production. Where the stomach contents and secretions from the liver and pancreas enter the duodenum, deficiency of mucus can cause irritation and, potentially, ulcers. Overstimulation of the oxyntic glands is not produced by the SNS. The hormone cholecystokinin, which is thought in part to control gastric emptying, is released in response to the pH, osmolality, and fatty acid composition of the chyme. The two largest salivary glands are called parotid glands. One is located in each cheek over the jaw in front of the ears. Inflammation of one or more of these glands is called parotitis.

11. A 42-year-old female client with a long-standing history of chronic nausea and vomiting but a near-insatiable appetite has had her symptoms attributed to an enzyme deficiency. Further diagnostic testing indicates that she has inadequate pancreatic enzyme levels and that her large appetite is due to a lack of enzyme control of food intake inhibition. In which of the following enzymes is the woman most likely deficient? A) Cholecystokinin B) Ghrelin C) Gastrin D) Secretin

Ans: A Feedback: Cholecystokinin is responsible for inhibiting food intake as well as stimulating pancreatic enzyme secretion. Ghrelin stimulates food intake, while gastrin stimulates gastric acid production, and secretin inhibits it.

13. Which of the following medications used in the treatment of peptic ulcers and gastroesophageal reflux binds to H2 receptors and blocks the action of histamine on parietal cells? A) Cimetidine (Tagamet) B) Levbid (hyoscyamine) C) Lotronex (alosetron) D) Nexium (esomeprazole)

Ans: A Feedback: Cimetidine is a H2 receptor blocker used to treat peptic ulcers and GERD. It binds to H2 receptors and blocks the action of histamine on parietal cells. Levbid and Lotronex are anticholinergics, while Nexium is a proton pump inhibitor that inhibits gastric acid secretion.

5. When the sympathetic nervous system is stimulated, the interstitial cells of Cajal, pacemaker cells of the GI tract, react by A) decreasing amplitude or abolishing the slow waves that control the spontaneous oscillations in membrane potentials. B) increasing the peristaltic motion of the GI tract, thereby causing explosive diarrhea. C) increasing the amount of secretions being entered into each segment of the intestinal tract. D) signaling the vagus nerve to slow down motility and increase absorption of water from the large intestine.

Ans: A Feedback: The interstitial cells of Cajal that are found in groups between the layers of smooth muscle tissue are hypothesized to function as the pacemakers. These cells display rhythmic, spontaneous oscillations in the membrane potentials, called slow waves, ranging in frequency from approximately 3 per minute in the stomach to 12 per minute in the duodenum. The vagus nerve responds to parasympathetic innervation. GI motility is enhanced because of increased vagal activity that could cause diarrhea.

1. A physician has ordered the measurement of a cardiac patient's electrolyte levels as part of the client's morning blood work. Which of the following statements best captures the importance of potassium in the normal electrical function of the patient's heart? A) Potassium catalyzes the metabolism of ATP, producing the gradient that results in electrical stimulation. B) Potassium is central to establishing and maintaining the resting membrane potential of cardiac muscle cells. C) The impermeability of cardiac cell membranes to potassium allows for action potentials achieved by the flow of sodium ions. D) The reciprocal movement of one potassium ion for one sodium ion across the cell membrane results in the production of an action potential.

Ans: B

1. In which of the following patient situations would a physician be most justified in preliminarily ruling out pericarditis as a contributing pathology to the patient's health problems? A) A 61-year-old man whose ECG was characterized by widespread T-wave inversions on admission but whose T waves have recently normalized B) A 77-year-old with diminished S3 and S4 heart tones, irregular heart rate, and a history of atrial fibrillation C) A 56-year-old obese man who is complaining of chest pain that is exacerbated by deep inspiration and is radiating to his neck and scapular ridge D) A 60-year-old woman whose admission blood work indicates elevated white cells, erythrocyte sedimentation rate, and C-reactive protein levels

Ans: B

1. Which of the following data would a clinician consider as most indicative of acute renal failure? A) Alterations in blood pH; peripheral edema B) Increased nitrogenous waste levels; decreased glomerular filtration rate (GFR) C) Decreased serum creatinine and blood urea nitrogen (BUN); decreased potassium and calcium levels D) Decreased urine output; hematuria; increased GFR

Ans: B

10. A male patient comes to the clinic asking to speak to a health care provider privately. He reveals that he had shared a needle/syringe with a prostitute (shooting up cocaine) and then had unprotected sex. Upon questioning, it was revealed that the patient had not had any immunization for hepatitis B. Which of the following medications would the nurse anticipate administering today to this patient? A) Tenofovir disoproxil fumarate plus emtricitabine B) Hepatitis B immunoglobulin (HBIG) C) Hepatitis C immunoglobulin (HCIG) D) Hepatitis A vaccine

Ans: B

10. When trying to educate a patient about the release of free radicals and the role they play in formation of atherosclerosis, which of the following statements is most accurate? A) The end result of oxidation is rupture of the plaque resulting in hemorrhage. B) Activated cells that release free radicals oxidize LDL, which is harmful to the lining of your blood vessels. C) Oxidized free radicals produce toxic metabolic waste that can kill liver cells. D) Activated cells roam in the vascular system looking for inflammatory cells to engulf.

Ans: B

10. Which of the following patients should the nurse be assessing for long QT syndrome? A) A 95-year-old patient with Alzheimer's who is having periods of apnea B) A 32-year-old male admitted for cocaine overdose with long history of illicit drug abuse C) A 56-year-old female admitted for total hysterectomy due to excessive bleeding and clotting D) A 68-year-old male who was in a car accident with sternal bruising and fractured femur

Ans: B

11. A 40-year-old man who uses heroin intravenously was diagnosed with hepatitis C (HCV) 1 year ago and is now considered to have chronic viral hepatitis. Which of the following statements by the client to his care provider would warrant correction? A) "I know the medications to treat this aren't fantastic, but at least there are some options for controlling the virus." B) "It's at least a bit reassuring that my liver isn't undergoing damage when I'm not experiencing symptoms." C) "Even though I'm sick, at least I won't feel sick most of the time." D) "I'm not looking forward to all the side effects of the drug treatments for my HCV, but I hope I don't end up needing a liver transplant."

Ans: B

11. A number of patients in an acute cardiac care unit of a hospital have diagnoses of impaired cardiac conduction. Which of the following patients is most deserving of immediate medical attention? A) A 46-year-old man whose cardiac telemetry shows him to be in ventricular tachycardia B) A 69-year-old woman who has entered ventricular fibrillation C) A 60-year-old man with premature ventricular contractions (PVC) and a history of atrial fibrillation D) A 60-year-old woman who has just been diagnosed with a first-degree AV block

Ans: B

11. A patient is experiencing impaired circulation secondary to increased systemic arterial pressure. Which of the following statements is the most relevant phenomenon? A) Increased preload due to vascular resistance B) High afterload because of backpressure against the left ventricle C) Impaired contractility due to aortic resistance D) Systolic impairment because of arterial stenosis

Ans: B

11. Because the associated nephropathy is an important cause of end-stage renal failure in children and adolescents, a toddler who has had an uncomplicated bout of urinary tract infection (UTI) should still be evaluated for A) urethrovesical reflux. B) vesicoureteral reflux. C) neurogenic bladder. D) detrusor muscle instability.

Ans: B

11. Which of the following clients' diagnostic blood work is most suggestive of chronic kidney disease (CKD)? A) A client with high pH; low levels of calcium; and low levels of phosphate B) A client with low vitamin D levels; low calcitriol levels; and elevated parathyroid hormone (PTH) levels C) A client with low bone density; low levels of calcium; and low levels of phosphate D) A client with low potassium levels; low calcitriol levels; and increased PTH levels

Ans: B

12. A 62-year-old female smoker is distraught at her recent diagnosis of small cell lung cancer (SCLC). How can her physician most appropriately respond to her? A) "I'm sure this is very hard news to hear, but be aware that with aggressive treatment, your chances of beating this are quite good." B) "This is very difficult to hear, I'm sure, and we have to observe to see if it spreads because that often happens." C) "I'm very sorry to have to give you this news; I'd like to talk to you about surgical options, however." D) "This is a difficult diagnosis to receive, but there is a chance that the cancer may go into remission."

Ans: B

12. Which of the following assessment findings of a newly admitted 30-year-old male client would be most likely to cause his physician to suspect polyarteritis nodosa? A) The man's blood work indicates polycythemia (elevated red cell levels) and leukocytosis (elevated white cells). B) The man's blood pressure is 178/102, and he has abnormal liver function tests. C) The man is acutely short of breath, and his oxygen saturation is 87%. D) The man's temperature is 101.9°F, and he is diaphoretic (heavily sweating).

Ans: B

12. Which of the following medications would the nurse anticipate being prescribed for the renal failure patient who has hyperphosphatemia? A) Vitamin D (calcitriol) B) Calcium carbonate C) Levothyroxine (Synthroid) D) Sensipar (Cinacalcet)

Ans: B

13. A patient is admitted to the outpatient diagnostic unit for further testing to identify the cause of the uncontrolled secondary hypertension. In preparation, the nurse should anticipate that which of the following diagnostic procedures will provide the most definitive diagnosis? A) Routine ultrasound of kidney B) Renal arteriography C) Echocardiography D) Serum creatinine level

Ans: B

15. A 34-year-old man who is an intravenous drug user has presented to the emergency department with malaise, abdominal pain, and lethargy. The health care team wants to rule out endocarditis as a diagnosis. Staff of the department would most realistically anticipate which of the following sets of diagnostics? A) CT of the heart, chest x-ray, and ECG B) Echocardiogram, blood cultures, and temperature C) ECG, blood pressure, and stress test D) Cardiac catheterization, chest x-ray, electrolyte measurement, and white cell count

Ans: B

15. A patient arrived at the emergency department 2 days after the development of "chest pressure" and "tightness" was treated with antacids thinking it was indigestion. His enzymes show a massive myocardial infarction (MI). Following angioplasty, the patient asks why so much muscle was damaged if only one vessel was blocked, the left circumflex. The nurse responds, A) "With any blockage in the heart, muscle damage always occurs." B) "If a major artery like the circumflex is occluded, the smaller vessels supplied by that vessel cannot restore the blood flow." C) "Since the circumflex artery supplies oxygenated blood flow to the posterior surface of the left ventricle, any amount of blockage will result in vital muscle tissue being lost." D) "When it comes to arteries in the heart, all vessels are equal, and any blockage causes a massive amount of damage that will not be restored."

Ans: B

15. A patient in the intensive care unit has a blood pressure of 87/39 and has warm, flushed skin accompanying his sudden decline in level of consciousness. The patient also has arterial and venous dilatation and a decrease in systemic vascular resistance. What is this client's most likely diagnosis? A) Hypovolemic shock B) Septic shock C) Neurogenic shock D) Obstructive shock

Ans: B

15. A patient is admitted for a relapse for sarcoidosis. Knowing this is usually caused by an inflammatory process, the nurse can anticipate administering A) a bronchodilator. B) a corticosteroid. C) aspirin. D) an albuterol inhaler.

Ans: B

15. Following electrophysiological testing that included ablation therapy, the nurse should be assessing the patient for which complication that may occur postprocedure? A) Complaints of nausea and spitting up bile-looking secretions along with stomach cramps B) Sudden onset of dysonea, tachypnea, and chest pain of a "pleuritic" nature (worsened by breathing) C) Bleeding from the nose that requires packing, excessive swallowing of mucus, and coughing D) Complaints of heart palpitations, frequent PVCs noted on monitor, and substernal chest pain

Ans: B

16. A 13-year-old boy has had a sore throat for at least a week and has been vomiting for 2 days. His glands are swollen, and he moves stiffly because his joints hurt. His parents, who believe in "natural remedies," have been treating him with various herbal preparations without success and are now seeking antibiotic treatment. Throat cultures show infection with group A streptococci. This child is at high risk for A) myocarditis. B) mitral valve stenosis. C) infective endocarditis. D) vasculitis.

Ans: B

16. During a routine physical examination of a 66-year-old woman, her nurse practitioner notes a pulsating abdominal mass and refers the woman for further treatment. The nurse practitioner is explaining the diagnosis to the client, who is unfamiliar with aneurysms. Which of the following aspects of the pathophysiology of aneurysms would underlie the explanation the nurse provides? A) Aneurysms are commonly a result of poorly controlled diabetes mellitus. B) Hypertension is a frequent modifiable contributor to aneurysms. C) Individuals with an aneurysm are normally asymptomatic until the aneurysm ruptures. D) Aneurysms can normally be resolved with lifestyle and diet modifications.

Ans: B

16. To reduce hepatic blood flow and decrease portal pressures in persons with cirrhosis, the nurse should be prepared to administer which of the following medications? A) Bevacizumab, an angiogenesis inhibitor B) Octreotide, a long-acting synthetic analog of somatostatin C) Filgrastim, granulocyte colony-stimulating factor (G-CSF) analog D) Diltiazem (Cardizem), a calcium channel blocker

Ans: B

16. Which of the following phenomena is most likely occurring during a child's alveolar stage of lung development? A) Terminal alveolar sacs are developing, and surfactant production is beginning. B) A single capillary network exists, and the lungs are capable of respiration. C) The conducting airways are formed, but respiration is not yet possible. D) Primitive alveoli are formed, and the bronchi and bronchioles become much larger.

Ans: B

17. A 16-year-old adolescent who received a kidney transplant at the age of 10 has recently developed a trend of increasing BP readings. Of the following list of medications, which may be the primary cause for the development of hypertension? A) Furosemide (Lasix) B) Cyclosporine (Sandimmune) C) Isotretinoin (Accutane) D) Hydrochlorothiazide (Hydrodiuril)

Ans: B

17. A nurse in an acute medical unit of a hospital has admitted a 62-year-old female from the emergency department who has been diagnosed with acute pyelonephritis. Which of the following statements most accurately conveys an aspect of the knowledge base that the nurse needs to perform adequate care and teaching? A) Most cases of acute pyelonephritis are attributable to poorly controlled hypertension. B) Flank pain, dysuria, and nausea and vomiting are likely assessment findings. C) The infection in the kidney is most likely a manifestation of a systemic infection. D) Imaging tests are likely to reveal scarring and deformation of the renal calices and pelvis.

Ans: B

18. A client has been diagnosed with mitral valve stenosis following his recovery from rheumatic fever. Which of the following teaching points would be most accurate to convey to the client? A) "The normal tissue that makes up the valve between the right sides of your heart has stiffened." B) "Your mitral valve isn't opening up enough for blood to flow into the part of your heart that sends blood into circulation." C) "Your heart's mitral valve isn't closing properly so blood is flowing backward in your heart and eventually into your lungs." D) "The valve between your left ventricle and left atria is infected and isn't allowing enough blood through."

Ans: B

18. When trying to explain to a new dialysis patient the movement of substances through the capillary pores, the nurse will explain that in the kidneys, the glomerular capillaries have A) no capillary openings since this would lead to extensive hemorrhage. B) small openings that allow large amounts of smaller molecular substances to filter through the glomeruli. C) large pores so that substances can pass easily through the capillary wall. D) endothelial cells that are joined by tight junctions that form a barrier to medication filtration.

Ans: B

19. A 3-year-old boy has developed croup following a winter cold. His care provider would recognize that which of the following microorganisms and treatments is most likely to be effective? A) Respiratory syncytial virus treated with intubation B) Parainfluenza virus treated with a mist tent and oxygen therapy C) Haemophilus influenza treated with appropriate antibiotics D) Staphylococcus aureus treated with bronchodilators and mist tent

Ans: B

19. A 41-year-old male client has presented to the emergency department with an acute onset of increased respiratory rate and difficulty breathing. STAT chest x-ray indicates diffuse bilateral infiltrates of his lung tissue, and ECG displays no cardiac dysfunction. What is this client's most likely diagnosis? A) Cor pulmonale B) Acute lung injury C) Pulmonary hypertension D) Sarcoidosis

Ans: B

19. A frantic mother brings her young child into the emergency department. She states that during the evening bath, she noticed a large mass in her child's abdomen. After diagnostic testing, the pediatrician tells the parents that their child has Wilms tumor, stage IV. After the doctor leaves the room, the parents ask the nurse, "What does this mean?" The nurse will respond, "Your child ('s) A) "has cancer in his stomach." B) "has cancer in the kidney that has spread most likely to his lungs." C) "will need to undergo surgery to remove both kidneys and then go on dialysis." D) "tumor can be easily treated with chemotherapy. We will start this soon."

Ans: B

19. A patient with a new automatic implantable cardioverter-defibrillator (AICD) asks the nurse what happens if he goes into that deadly heart rhythm again. The nurse will base her response knowing that the AICD will A) periodically fire just to test for lead placement and battery life. B) respond to ventricular tachyarrhythmia by delivering a shock within 10 to 20 seconds of its onset. C) use radiofrequency energy to deliver an electrical shock through the site where the lethal rhythm originates. D) remove scar tissue and aneurysm during placement of electrodes and then will shock if paradoxical ventricular movement is located.

Ans: B

19. An elderly patient arrives to the health care provider's office complaining of a "sore" that would not heal on his lower leg. Upon assessment, the nurse finds thin, shiny, bluish brown pigmented desquamative skin. It is located medially over the lower leg. The nurse will educate the patient that the usual treatment is A) hydrotherapy to facilitate improvement in circulation. B) compression therapy to help facilitate blood flow back to the vena cava. C) initiation of Coumadin therapy to maintain an INR of 2 to 3 above norm. D) long-term antibiotic therapy to facilitate healing of the wound.

Ans: B

2. A 22-year-old female with a history of intermittent flank pain, repeated UTIs, and hematuria has been diagnosed with autosomal dominant polycystic kidney disease (ADPKD). Which of the following phenomena has most likely contributed to the development of this diagnosis? A) UTIs coupled with an impaired immune response have caused her ADPKD. B) She has inherited a tendency for epithelial cells in her tubules to proliferate inappropriately. C) Severe hypertension and portal hypertension are likely precursors. D) She has inherited undersized kidneys that are prone to calculi formation.

Ans: B

20. During a family picnic, a relative of a nurse asks what he should do if there is blood in his urine and some pain in his lower abdomen. The best advice the nurse could give this family member would be for him to A) go to the emergency room right away. B) get an appointment with his family doctor. C) wait and see if it goes away without treatment. D) increase his intake of cranberry juice and other fluids.

Ans: B

20. Which of the following clinical findings among older adults is most unlikely to warrant further investigation and possible treatment? A) An 81-year-old male's serum creatinine level has increased sharply since his last blood work. B) A 78-year-old female's GFR has been steadily declining over several years. C) A 90-year-old female's blood urea nitrogen (BUN) is rising. D) An 80-year-old male whose urine dipstick reveals protein is present.

Ans: B

3. A 55-year-old male who is beginning to take a statin drug for his hypercholesterolemia is discussing cholesterol and its role in health and illness with his physician. Which of the following aspects of hyperlipidemia would the physician most likely take into account when teaching the patient? A) Hyperlipidemia is a consequence of diet and lifestyle rather than genetics. B) HDL cholesterol is often characterized as being beneficial to health. C) Cholesterol is a metabolic waste product that the liver is responsible for clearing. D) The goal of medical treatment is to eliminate cholesterol from the vascular system.

Ans: B

3. An ECG technician is placing leads on a patient who has presented to the emergency department with a sudden onset of chest pain. The technician would recognize which of the following facts about the placement of leads and the achievement of a clinically accurate ECG? A) The electrical potential recorded by a lead on an extremity will vary significantly depending on where the lead is placed on the extremity. B) The chest leads measure electrical activity on the horizontal plane, while limb leads measure it on the vertical plane. C) Limb leads measure the electrical activity of the heart indirectly through the activity of adjacent skeletal muscle. D) A total of 12 chest leads are necessary to attain the most accurate ECG.

Ans: B

4. The nurse working in the ICU knows that chronic elevation of left ventricular end-diastolic pressure will result in the patient displaying which of the following clinical manifestations? A) Chest pain and intermittent ventricular tachycardia B) Dyspnea and crackles in bilateral lung bases C) Petechia and spontaneous bleeding D) Muscle cramping and cyanosis in the feet

Ans: B

5. A 77-year-old patient with a history of coronary artery disease and heart failure has arrived in the emergency room with a rapid heart rate and feeling of "impending doom." Based on pathophysiologic principles, the nurse knows the rapid heart rate could A) decrease renal perfusion and result in the development of ascites. B) be a result of catecholamines released from SNS that could increase the myocardial oxygen demand. C) desensitize the -adrenergic receptors leading to increase in norepinephrine levels. D) prolong the electrical firing from the SA node resulting in the development of a heart block.

Ans: B

5. A group of novice nursing students are learning how to manually measure a client's blood pressure using a stethoscope and sphygmomanometer. Which of the following statements by students would the instructor most likely need to correct? A) "I'll inflate the cuff around 30 mm Hg above the point at which I can't palpate the client's pulse." B) "If my client's arm is too big for the cuff, I'm going to get a BP reading that's artificially low." C) "The accuracy of the whole process depends on my ability to clearly hear the Korotkoff sounds with the bell of my stethoscope." D) "With practice, my measurement of clients' blood pressures with this method will be more accurate than with automated machines."

Ans: B

5. A pneumonia that occurs 48 hours or more after admission to the hospital is considered A) community-acquired pneumonia. B) hospital-acquired pneumonia. C) viral pneumonia. D) immunocompromised pneumonia.

Ans: B

5. Which of the following teaching points would be most appropriate for a group of older adults who are concerned about their cardiac health? A) "People with plaque in their arteries experience attacks of blood flow disruption at seemingly random times." B) "The plaque that builds up in your heart vessels obstructs the normal flow of blood and can even break loose and lodge itself in a vessel." C) "Infections of any sort are often a signal that plaque disruption is in danger of occurring." D) "The impaired function of the lungs that accompanies pneumonia or chronic obstructive pulmonary disease is a precursor to plaque disruption."

Ans: B

6. A 51-year-old female client who is 2 days postoperative in a surgical unit of a hospital is at risk of developing atelectasis as a result of being largely immobile. Which of the following teaching points by her nurse is most appropriate? A) "Being in bed increases the risk of fluid accumulating between your lungs and their lining, so it's important for you to change positions often." B) "You should breathe deeply and cough to help your lungs expand as much as possible while you're in bed." C) "Make sure that you stay hydrated and walk as soon as possible to avoid us having to insert a chest tube." D) "I'll proscribe bronchodilator medications that will help open up your airways and allow more oxygen in."

Ans: B

6. A 60-year-old man has been diagnosed with renal calculi after repeated episodes of excruciating flank pain in recent weeks. The man states that, "I don't know how this could happen to me, since I'm so careful about eating a healthy diet." What is the most appropriate response to the man's statement? A) "Your diet may have played a part in this, but in fact, genetics are likely primarily to blame." B) "What you eat can influence your risk of stone formation, but many other factors like hormones and your metabolism are involved." C) "You likely don't need to change your diet, but now that you have stones in one kidney, you're at very high risk of growing them in the other kidney." D) "Your diet might be normally healthy, but high intake of normally beneficial minerals like calcium and magnesium can lead to stones."

Ans: B

6. A nurse is providing care for an older, previously healthy adult male who has been diagnosed today with pneumococcal pneumonia. Which of the following signs and symptoms is the nurse most likely to encounter? A) The man will be hypotensive and febrile and may manifest cognitive changes. B) The patient will have a cough producing clear sputum, and he will have faint breath sounds and fine crackles. C) The patient will have copious bloody sputum and diffuse chest pain and may lose his cough reflex. D) The patient will lack lung consolidation and will have little, if any, sputum production.

Ans: B

7. A client with a newborn infant is also the caregiver for her 75-year-old mother, who lives with them and who has diabetes. The client requests pneumonia vaccinations for her entire household. Which vaccine is most likely to be effective for the baby? A) Since the baby's immune system is mature at birth, regular vaccine is appropriate. B) There is no effective vaccine for newborn infants. C) The 23-valent vaccine will be effective. D) No vaccine is necessary for the baby if the nursing mother is immunized.

Ans: B

7. When advising a morbidly obese patient about the benefits of weight reduction, which of the following statements would be most accurate to share? A) "All you need to do is stop drinking sodas and sugary drinks." B) "A 10 lb loss of weight can produce a decrease in blood pressure." C) "An increased 'waist-to-hip' ratio can lead to too much pressure on the liver and intestines." D) "If your leptin (hormone) level is too low, you are at increased risk for developing high BP."

Ans: B

7. Which of the following pain descriptions would lead the nurse to suspect the client is experiencing ureteral colic? A) Right upper quadrant pain that worsens with deep breaths and palpation B) Excruciating pain in the flank and upper outer quadrant of the abdomen that radiates to the bladder area C) Pain described as "fire poking in their side," pulsating with every heart beat but decreases when in fetal position D) Perineal pain that increases when urinating and then lessens until the time to urinate again

Ans: B

8. A medical student is working with a 61-year-old male client in the hospital who has presented with a new onset of atrial fibrillation. Which of the following courses of treatment will the student most likely expect the attending physician to initiate? A) Immediate cardioversion followed by surgery to correct the atrial defect B) Anticoagulants and beta-blockers to control rate C) Antihypertensives and constant cardiac monitoring in a high acuity unit D) Diuretics, total bed rest, and cardioversion if necessary

Ans: B

8. A nurse is collecting a urine specimen prior to measuring the albumin level in a client's urine. A colleague questions the rationale for the test, stating, "I thought albumin was related to liver function, not kidney function." How can the nurse best respond to this statement? A) "Urine should normally be free of any proteins, and albumin is one of the more common proteins to be excreted in chronic renal failure." B) "Urine albumin levels are useful for diagnosing diabetic kidney disease." C) "A urine dipstick test will tell us exactly how much albumin is being spilled by the client's kidneys." D) "A urine test for albumin allows us to estimate the client's GFR quite accurately."

Ans: B

8. Four weeks after returning from a tropical vacation, a 40-year-old man has presented to the emergency department with malaise, nausea, and "yellow eyes." Serology has confirmed a diagnosis of hepatitis A (HAV), to the shock of the client. What teaching is most appropriate for this client? A) "You can expect these symptoms to disappear after about 2 months, but you'll be a carrier of the disease indefinitely." B) "A vaccine before your trip would have prevented this, but be assured your body will rid itself of the virus in time." C) "You likely came in contact with blood or body fluids at some point, and you'll have to ensure no one is subsequently exposed to your own blood or body fluids." D) "You likely got this by way of what we call the 'fecal-oral' route; you will have chronic hepatitis now, but the symptoms can be controlled with medication."

Ans: B

9. A 53-year-old woman with a history of chronic alcohol abuse but without visible jaundice comes to the clinic complaining of nausea and weakness. She admits to taking acetaminophen for persistent headaches but denies exceeding the recommended daily dose; she has not taken any other medications. She is suspected of having acetaminophen toxicity. Which of the following diagnostic test findings would implicate a different cause of her symptoms? A) Normal serum acetaminophen level B) Elevated serum HBsAg level C) Evidence of steatosis on liver biopsy tissue sample D) Hypoglycemia

Ans: B

9. A nurse is administering morning medications to a number of patients on a medical unit. Which of the following medication regimens is most suggestive that the patient has a diagnosis of heart failure? A) Antihypertensive, diuretic, antiplatelet aggregator B) Diuretic, ACE inhibitor, beta-blocker C) Anticoagulant, antihypertensive, calcium supplement D) Beta-blocker, potassium supplement, anticoagulant

Ans: B

9. An 81-year-old female client of a long-term care facility has a history of congestive heart failure. The nurse practitioner caring for the client has positioned her sitting up at an angle in bed and is observing her jugular venous distention. Why is jugular venous distention a useful indicator for the assessment of the client's condition? A) Increased cardiac demand causes engorgement of systemic blood vessels, of which the jugular vein is one of the largest. B) Blood backs up into the jugular vein because there are no valves at the point of entry into the heart. C) Peripheral dilation is associated with decreased stroke volume and ejection fraction. D) Heart valves are not capable of preventing backflow in cases of atrial congestion.

Ans: B

20. Which of the following statements best captures an aspect of the process of fat digestion and absorption? A) Ingested triglycerides are broken down into absorbable form by gastric lipase. B) Triglycerides are digested with the aid of bile salts. C) Long-chain fatty acids are absorbed directly into the portal blood. D) Stool is not excreted until all fat is absorbed.

Ans: B Feedback: After breakdown, by pancreatic lipase, triglycerides are absorbed primarily in the upper jejunum. Long-chain fatty acids are absorbed less easily than medium-chain triglycerides, while stool often contains a certain amount of fat.

19. A mom asks her neighbor, a nurse, why every time she takes her daughter (10 years old) out for ice cream she comes home with a stomachache and then experiences a bout of diarrhea. The nurse is thinking that this girl is experiencing A) anxiety about increasing too many calories. B) a deficiency of lactase. C) gallbladder disease. D) premature peptic ulcer formation.

Ans: B Feedback: People with a deficiency of lactase, the enzyme that breaks down lactose, experience diarrhea when they drink milk or eat dairy products. Doubtful the child is anxious about the calories in ice cream. Gallbladder disease s/s usually occur in relation to high saturated fat intake. Peptic ulcer s/s include bloating, vomiting blood, foul coffee ground stools, etc.

6. A gastroenterologist is teaching a group of medical students about the enteric nervous system in preparation for a consult on client who has suffered a spinal cord injury. Which of the physician's teaching points is most accurate? A) "The myenteric plexus is responsible for controlling the function of each segment of the intestinal tract." B) "The enteric nervous system is made up of the myenteric and submucosal plexuses; these are located in the wall of the GI tract." C) "Sympathetic innervation of much of the GI tract occurs by way of the vagus nerve." D) "Parasympathetic stimulation blocks the release of the excitatory neuromediators and inhibits GI motility."

Ans: B Feedback: The enteric nervous system consists of the myenteric and submucosal plexuses, which are located within the wall of the gastrointestinal tract. The myenteric plexus is responsible for controlling overall function along the entire length of the gut, while the vagus nerve provides parasympathetic, not sympathetic, innervation. Sympathetic simulation lessens excitatory neuromediators and inhibits GI motility.

. 5. A short, nonsmoking 44-year-old male presents to the emergency room with left-sided chest pain and a cough. He states that the pain started abruptly and worsens with deep breathing and coughing. He denies recent injury. Assessment includes shallow respirations with a rate of 36, normal breath sounds, and no cyanosis. Which condition is most likely causing his symptoms? A) Myocardial infarction B) Spontaneous pneumothorax C) Pleuritis related to infection D) Obstructive atelectasis

Ans: C

1. A 74-year-old man is being assessed by a nurse as part of a weekly, basic health assessment at the long-term care facility where he resides. His blood pressure at the time is 148/97 mm Hg, with a consequent pulse pressure of 51 mm Hg. The nurse would recognize that which of the following is the most significant determinant of the resident's pulse pressure? A) Blood volume, resistance, and flow B) The cardiac reserve or possible increase in cardiac output over normal resting level C) The amount of blood that his heart ejects from the left ventricle during each beat D) The relationship between total blood volume and resting heart rate

Ans: C

1. If a virus has caused inflammation resulting in endothelial dysfunction, an excessive amount of endothelins in the blood can result in A) arterial wall weakening resulting in aneurysm formation. B) release of excess fatty plaque causing numerous pulmonary emboli. C) contraction of the underlying smooth muscles within the vessels. D) overproduction of growth factors resulting in new vessel production.

Ans: C

1. When explaining the role of liver Kupffer cells to a group of nursing students, which of the following statements about the function of these cells is most accurate? A) The primary function of Kupffer cells is to secrete bile. B) These cells are the functional unit of the liver and are responsible for all liver secretions. C) The cells are capable of removing and phagocytizing old and defective blood cells. D) The role of the Kupffer cells is to provide at least 50% of cardiac output each minute to each lobular of the liver.

Ans: C

10. A 24-year-old college student has presented to the campus medical clinic with complaints of frequent, burning urination and has, subsequent to urinalysis, been diagnosed with an acute lower urinary tract infection (UTI) caused by E. coli. What teaching will the clinician most likely provide to the student? A) "This should likely resolve itself if you drink a lot of water and especially cranberry or blueberry juice." B) "Unfortunately, the bacteria causing your infection is no longer responsive to antibiotics, but there are alternative treatments that we can use." C) "Many of these bacteria are now resistant to some antibiotics, but I will take that into account when I choose which antibiotic to prescribe." D) "This likely shows that you have some sort of obstruction in your urinary system, so when that is treated your UTI will likely resolve as well."

Ans: C

10. A 78-year-old man has been experiencing nocturnal chest pain over the last several months, and his family physician has diagnosed him with variant angina. Which of the following teaching points should the physician include in his explanation of the man's new diagnosis? A) "I'll be able to help track the course of your angina through regular blood work that we will schedule at a lab in the community." B) "With some simple lifestyle modifications and taking your heparin regularly, we can realistically cure you of this." C) "I'm going to start you on low-dose aspirin, and it will help greatly if you can lose weight and keep exercising." D) "There are things you can do to reduce the chance that you will need a heart bypass, including limiting physical activity as much as possible."

Ans: C

10. As part of their orientation to a cardiac care unit, a group of recent nursing graduates is receiving a refresher in cardiac physiology from the unit educator. Which of the following teaching points best captures a component of cardiac function? A) "Efficient heart function requires that the ventricles do not retain any blood at the end of the cardiac cycle." B) "Recall that the heart sounds that we listen to as part of our assessments are the sounds of the myocardium contracting." C) "The diastolic phase is characterized by relaxation of ventricles and their filling with blood." D) "Aortic pressure will exceed ventricular pressure during systole."

Ans: C

10. When educating a student who lives in a crowded apartment and diagnosed with tuberculosis, the college school nurse will emphasize, A) "Once your fever goes away, you can stop taking the streptomycin injection." B) "If isoniazid makes you nauseous, we can substitute something milder." C) "To destroy this bacterium, you must strictly adhere to a long-term drug regimen." D) "You will have to wear an N95 mask while on campus at all times."

Ans: C

10. Which of the following residents of a long-term care facility is most likely to be exhibiting the signs and symptoms of chronic obstructive pulmonary disease (COPD)? A) A 79-year-old lifetime smoker who is complaining of shortness of breath and pain on deep inspiration B) An 81-year-old smoker who has increased exercise intolerance, a fever, and increased white blood cells C) An 81-year-old male who has a productive cough and recurrent respiratory infections D) An 88-year-old female who experiences acute shortness of breath and airway constriction when exposed to tobacco smoke

Ans: C

11. A nurse is providing care for a number of older clients on a restorative care unit of a hospital. Many of the clients have diagnoses or histories of hypertension, and the nurse is responsible for administering a number of medications relevant to blood pressure control. Which of the following assessments would the nurse be most justified in eliminating during a busy morning on the unit? A) Checking the recent potassium levels of a client receiving an ACE inhibitor B) Measuring the heart rate of a client who takes a -adrenergic blocker C) Measuring the pulse of a client taking an ACE inhibitor D) Noting the sodium and potassium levels of a client who is receiving a diuretic

Ans: C

11. The initial medical management for a symptomatic patient with obstructive hypertrophic cardiomyopathy (HCM) would be administering a medication to block the effects of catecholamines. The nurse will anticipate administering which of the following medications? A) Lisinopril, an ACE inhibitor B) Lasix, a diuretic C) Propranolol, a -adrenergic blocker D) Lanoxin, an inotropic

Ans: C

12. A 22-year-old male is experiencing hypovolemic shock following a fight in which his carotid artery was cut with a broken bottle. What immediate treatments are likely to most benefit the man? A) Resolution of compensatory pulmonary edema and heart arrhythmias B) Infusion of vasodilators to foster perfusion and inotropes to improve heart contractility C) Infusion of normal saline or Ringer lactate to maintain the vascular space D) Administration of oxygen and epinephrine to promote perfusion

Ans: C

12. A 71-year-old man is slated for pacemaker insertion for treatment of a third-degree AV block. The man's nurse has been educating him about his diagnosis and treatment and answering the numerous questions he has about his health problem. Which of the following teaching points should the nurse include in this patient teaching? A) "This is almost certainly a condition that you were actually born with, but that is just now becoming a serious problem." B) "Because the normal electrical communication in lacking, the bottom parts of your heart are beating especially fast to compensate for inefficiency." C) "The root problem is that the top chambers of your heart and the bottom chambers of your heart aren't coordinating to pump blood efficiently." D) "If left untreated, this would have put you at great risk of stroke or heart attack."

Ans: C

12. In the ICU, the nurse hears an emergency cardiac monitor go off. The nurse looks at the telemetry and notices the patient has gone into ventricular tachycardia. The nurse will likely assess for signs/symptoms of A) development of hypertension with BP 190/98. B) oxygen deprivation with O2 saturation decreasing to approximately 90%. C) decreasing cardiac output due to less ventricular filling time. D) increasing cardiac index by correlating the volume of blood pumped by the heart with an individual's body surface area.

Ans: C

13. A 30-year-old woman presents at a hospital after fainting at a memorial service, and she is diagnosed as being in neurogenic shock. Which of the following signs and symptoms is she most likely to display? A) Faster than normal heart rate B) Pain C) Dry and warm skin D) Increased thirst

Ans: C

13. A 72-year-old woman with a recent onset of syncopal episodes has been referred to a cardiologist by her family physician. As part of the client's diagnostic workup, the cardiologist has ordered her to wear a Holter monitor for 24 hours. Which of the following statements best captures an aspect of Holter monitoring? A) A Holter monitor is preferable to standard ECG due to its increased sensitivity to cardiac electrical activity. B) The primary goal is to allow the cardiologist to accurately diagnose cardiomyopathies. C) Accurate interpretation of the results requires correlating the findings with activity that the woman was doing at the time of recording. D) Holter monitors are normally set to record electrical activity of the heart at least once per hour.

Ans: C

13. To maintain hematocrit levels in people with kidney failure, the nurse should be prepared to A) arrange for frequent blood transfusions in an outpatient clinic. B) administer iron dextran intravenously. C) administer a subcutaneous injection of recombinant human erythropoietin (rhEPO). D) administer prenatal vitamins twice a day.

Ans: C

14. A 70-year-old male client presents to the emergency department complaining of pain in his calf that is exacerbated when he walks. His pedal and popliteal pulses are faintly palpable, and his leg distal to the pain is noticeably reddened. The nurse knows that the client is likely experiencing which of the following medical diagnosis/possible treatment plans listed below? A) Acute arterial occlusion that will be treated with angioplasty B) Raynaud disease that will require antiplatelet medications C) Atherosclerotic occlusive disease necessitating thrombolytic therapy D) Giant cell temporal arteritis that will be treated with corticosteroids

Ans: C

14. A nurse educator is performing client education with a 51-year-old man who has been recently diagnosed with chronic kidney disease. Which of the following statements by the client would the nurse most likely want to correct or clarify? A) "I'll be prone to anemia, since I'm not producing as much of the hormone that causes my bones to produce red blood cells." B) "My heart rate might go up because of my kidney disease, and my blood might be a lot thinner than it should be." C) "My kidney problems increase my chance of developing high blood pressure or diabetes." D) "I'll have a risk of either bleeding too easily or possibly clotting too quickly, though dialysis can help minimize these effects."

Ans: C

14. A physical assessment of a 28-year-old female patient indicates that her blood pressure in her legs is lower than that in her arms and that her brachial pulse is weaker in her left arm than in her right. In addition, her femoral pulses are weak bilaterally. Which of the following possibilities would her care provider most likelysuspect? A) Pheochromocytoma B) Essential hypertension C) Coarctation of the aorta D) An adrenocortical disorder

Ans: C

14. Analysis has shown that a client's right atrial pressure is 30 mm Hg. What is the most likely conclusion that the client's care team will draw from this piece of data? A) The result is likely normal and gravity dependent given the lack of valves in thoracic and central veins. B) The pressure is insufficient to provide adequate stroke volume and cardiac output. C) The pressure is excessive given that the right atrium should be at atmospheric pressure. D) Pressure pulsations are likely to be undetectable given the low atrial pressure.

Ans: C

14. Which of the following clinical findings would be most closely associated with a client who has interstitial lung disease in comparison to chronic obstructive pulmonary disease (COPD)? A) Audible wheezing on expiration B) Reduced expiratory flow rates C) Decreased tidal volume D) Normal forced expiratory volume

Ans: C

15. A 25-year-old Asian American man arrives in the emergency room in a panic. Except for a bout with bronchitis a week earlier, he has been healthy his entire life; today he has blood in his urine. What disease has likely caused of his hematuria and how should it be treated? A) Goodpasture syndrome and will be treated with plasmapheresis and immunosuppressive therapy B) Membranous glomerulonephritis and should be treated with corticosteroids C) Immunoglobulin A nephropathy and may be advised to use omega-3 fatty acids to delay progression of disease D) Kimmelstiel-Wilson syndrome and should be treated with medication to control high blood pressure

Ans: C

15. A patient with pancreatic cancer is admitted for portal hypertension in which he is symptomatic with ascites. Following paracentesis and removal of 7.5 L of ascitic fluid, the nurse should anticipate that the physician will order which of the following medications to assist in maintaining an effective circulating fluid volume? A) Bumetanide (Bumex) B) Furosemide (Lasix) C) Albumin (human) 5% D) Epogen (epoetin alfa)

Ans: C

15. During a prenatal education class, an expectant mother tells the group about a friend whose blood pressure became so high during pregnancy that she had to be admitted to hospital. Which of the following statements should the nurse include in response to this? A) "A large increase in blood pressure is a normal part of the changes in blood circulation that accompany pregnancy." B) "By avoiding salt, staying active, and minimizing weight gain, you can prevent this during your pregnancy." C) "Essentially, experts don't really know why so many pregnant women develop high blood pressure." D) "I'm sure this was hard for your friend, but rest assured that it won't affect your baby even if it affects you."

Ans: C

15. The neonatal ICU nurse is aware that type II alveolar cells produce surfactant, and they usually develop at how many weeks of gestation? A) 17 to 18 weeks B) 19 to 20 weeks C) 24 to 28 weeks D) 34 to 38 weeks

Ans: C

15. The nurse assessing a renal failure patient for encephalopathy caused by high uremic levels may observe which of the following clinical manifestations? A) Severe chest pain with pericardial friction rub on auscultation B) Stiff immobile joints and contractures C) Loss of recent memory and inattention D) Pruritus with yellow hue to skin tone

Ans: C

16. A formerly normotensive woman, pregnant for the first time, develops hypertension and headaches at 26 weeks' gestation. Her blood pressure is 154/110 mm Hg, and she has proteinuria. What other lab tests should be ordered for her? A) Plasma angiotensin I and II and renin B) Urinary sodium and potassium C) Platelet count, serum creatinine, and liver enzymes D) Urinary catecholamines and metabolites

Ans: C

17. A 3-year-old child with right-sided heart failure has been admitted for worsening of his condition. Which of the following assessments would be considered one of the earliest signs of systemic venous congestion in this toddler? A) Breathlessness with activity B) Excessive crying C) Enlargement of the liver D) Increased urine output

Ans: C

17. In which of the following patients is the emergency department staff most likely to suspect an abdominal aortic aneurysm? A) A 60-year-old client with diminished oxygen saturation, low red blood cell levels, and pallor B) A 70-year-old woman with jugular venous distention, shortness of breath, and pulmonary edema C) A 66-year-old client with facial edema, cough, and neck vein distention D) An 81-year-old man with acute cognitive changes as well as difficulty in speaking and swallowing

Ans: C

17. On a routine physical exam visit, the physician mentions that he hears a new murmur. The patient gets worried and asks, "What does this mean?" The physician responds, A) "It would be caused by stress. Let's keep our eye on it and see if it goes away with your next visit." B) "This could be caused by an infection. Have you been feeling well the past few weeks?" C) "One of your heart valves is not opening properly. We need to do an echocardiogram to see which valve is having problem." D) "This may make you a little more fatigued than usual. Let me know if you start getting dizzy or light-headed."

Ans: C

18. A 34-year-old man has been taking up to 2400 mg of ibuprofen per day following a motor vehicle accident several months ago and consequent chronic pain. He has recently been diagnosed with chronic analgesic nephritis as a result of his high analgesic intake. The man is surprised at the diagnosis stating, "I thought that taking too many drugs hurts your liver if anything, not your kidneys." What is the most appropriate response to the man's statement? A) "Your liver does perform most of the detoxification in your body, but your kidneys can perform this role if the liver is unable to." B) "High drug intake can cause your kidneys to be very vulnerable to infections, which is likely what happened in your case." C) "Your kidneys are vulnerable to damage because of how much blood flows through them and the fact that they break down many drugs." D) "It is very rare for someone as young as yourself to have kidney damage like this; usually only older people are vulnerable to kidney damage from drugs."

Ans: C

18. Following several days of intermittent upper right quadrant pain, a 29-year-old obese, Native American woman has been diagnosed with cholelithiasis. The nurse at the clinic has taught the client about the pathophysiology and contributing factors to her health problem, as well as some of the likely treatment options. Which of the following statements by the client demonstrates a sound understanding of her diagnosis? A) "All in all, I guess this is a result of the fact that I've been eating a diet too high in cholesterol for too long." B) "Several factors like my genetics and gender may have contributed to this, but I'm glad that medications can cure it." C) "This explains why my skin was yellow-tinged lately and why I had those pains that spread to my upper back and right shoulder." D) "I suppose the fever and vomiting I had this week was probably a sign of my gallstones too."

Ans: C

19. A 51-year-old patient with a history of alcohol abuse and liver disease has low serum levels of albumin and presents with ascites (excess fluid in his peritoneal space) and jaundice. A health care professional would recognize that which of the following processes is most likely underlying his health problems? A) Low albumin is contributing to excess hydrostatic pressure and inappropriate fluid distribution. B) Low albumin is inducing hypertension and increased filtration of fluid into interstitial spaces. C) Insufficient albumin is causing insufficient absorption of fluid into the capillaries. D) Low albumin contributing to an inability to counter gravitational effects.

Ans: C

2. A 51-year-old male has been diagnosed with alcohol-induced liver disease. He admits to the nurse providing his care that, "I know what the lungs do, and I know what the heart does, but honestly I have no idea what the liver does in the body." Which of the following statements would best underlie the explanation that the nurse provides? A) The liver is responsible for the absorption of most dietary nutrients as well as the production of growth hormones. B) The liver contributes to the metabolism of ingested food and provides the fluids that the GI tract requires. C) The liver metabolizes most components of food and also cleans the blood of bacteria and drugs. D) The liver maintains a balanced level of electrolytes and pH in the body and stores glucose, minerals, and vitamins.

Ans: C

2. A nurse will be providing care for a female patient who has a diagnosis of heart failure that has been characterized as being primarily right sided. Which of the following statements best describes the presentation that the nurse should anticipate? The client A) has a distended bladder, facial edema, and nighttime difficulty breathing. B) complains of dyspnea and has adventitious breath sounds on auscultation (listening). C) has pitting edema to the ankles and feet bilaterally, decreased activity tolerance, and occasional upper right quadrant pain. D) has cyanotic lips and extremities, low urine output, and low blood pressure.

Ans: C

2. At 4 AM, the hemodynamic monitor for a critically ill client in the intensive care unit indicates that the client's mean arterial pressure is at the low end of the normal range; at 6 AM, the client's MAP has fallen definitively below normal. The client is at risk for A) pulmonary hypertension. B) left ventricular hypertrophy. C) organ damage and hypovolemic shock. D) orthostatic hypotension.

Ans: C

2. In which of the following situations would blood most likely rapidly relocate from central circulation to the lower extremities? A) A client undergoes a stress test on a treadmill. B) A client does isotonic exercises in a wheelchair. C) A client is helped out of bed and stands up. D) A client reclines from a sitting to supine position.

Ans: C

20. A 70-year-old male with a 40 pack-year history of smoking and long-standing non-insulin-dependent diabetes has been diagnosed with pancreatic cancer. Which of the following teaching points should the physician provide? A) "While this is indeed serious, you should know that you have a good chance of beating this disease with appropriate treatment." B) "Most likely your pattern of high alcohol intake over the years contributed to your cancer." C) "You will likely be facing surgery in the near future, but know that this is very unlikely to eliminate your cancer." D) "I know it may seem trivial at this point, but the levels of pain that accompany cancer of the pancreas are normally quite low."

Ans: C

20. Assuming that they have not responded to drug therapy, which of the following clients is likely to be the best candidate for surgical cardiac ablation? A) A 62-year-old woman with peripheral vascular disease who has experienced multiple episodes of torsade des pointes B) A 75-year-old man with diabetes but no previous heart disease that suddenly develops syncope due to sick sinus syndrome C) A 46-year-old man with unstable angina and a history of myocardial infarction who is found to have long QT syndrome and episodes of frequent ventricular arrhythmias D) A 22-year-old woman with an atrial septal defect who has recurrent paroxysmal atrial flutter with rapid ventricular rate associated with her caffeine intake.

Ans: C

20. While intubated for surgery, a patient has inadvertently had his vagus nerve stimulated. What effect would the surgical team expect to observe? A) Decreased vascular perfusion due to parasympathetic stimulation B) Decreased heart rate, contractility, and afterload C) Decreased heart rate as a result of parasympathetic innervation of the heart D) Decreased heart rate as a result of impaired acetylcholine reuptake

Ans: C

21. A patient in the emergency department is experiencing a massive stroke with extremely low blood flow to the brain exhibited by a BP less than 60 mm Hg. The nurse suddenly notes there is a sharp rise in the BP to 250 mm Hg. This high BP lasts about 5 minutes, and then the BP drops sharply again. The pathophysiologic principle behind this is likely due to the A) activation of the autonomic nervous system. B) release of mineralocorticoids. C) CNS ischemic response. D) protective homeostatic mechanism.

Ans: C

22. A nurse who works on a pediatric cardiology unit of a hospital is providing care for an infant with a diagnosis of tetralogy of Fallot. Which of the following pathophysiologic results should the nurse anticipate? A) There is a break in the normal wall between the right and left atria that results in compromised oxygenation. B) The aortic valve is stenotic, resulting in increased afterload. C) Blood outflow into the pulmonary circulation is restricted by pulmonic valve stenosis. D) The right ventricle is atrophic as a consequence of impaired myocardial blood supply.

Ans: C

3. A child with rhinosinusitis should be monitored for complications. Which of the following assessment findings would alert the nurse that a complication is developing? A) Purulent nasal discharge B) Temperature of 100.8°F C) Periorbital edema D) Complaints of headache

Ans: C

3. One of the most reliable predictors for worsening autosomal dominant polycystic kidney disease is A) serum creatinine levels. B) blood urea nitrogen (BUN) level. C) urine albumin excretion (UAE). D) urine specific gravity.

Ans: C

3. The clinical nurse educator on a nephrology unit of a large, urban hospital is orientating recent nursing graduates to the unit. Which of the following teaching points about acute tubular necrosis (ATN) should the educator include in the orientation session? A) "The cardinal signs of ATN are oliguria and retention of potassium, creatinine, and sulfates." B) "Ureteral and bladder outlet obstructions are often contributors to ATN." C) "Trauma, burns, and major surgery are common precursors to ATN." D) "Tubular epithelial cells are sensitive to ischemia and toxins, and damage is irreversible."

Ans: C

3. Which of the following phenomena would be most likely to accompany increased myocardial oxygen demand (MVO2)? A) Inadequate ventricular end-diastolic pressure B) Use of calcium channel blocker medications C) Increased aortic pressure D) Ventricular atrophy

Ans: C

4. The cardiologist just informed a patient that he has a reentry circuit in the electrical conduction system in his heart. This arrhythmia is called Wolff-Parkinson-White (WPW) syndrome. After the physician has left the room, the patient asks the nurse to explain this to him. Which of the following statements most accurately describes what is happening? A) "This means that the SA node (which is the beginning of your heart's electrical system) has been damaged and is no longer functioning normal." B) "You must have a large clot in one of your arteries that supply oxygenated blood to the special conduction cells in your heart." C) "There is an extra, abnormal electrical pathway in the heart that leads to impulses traveling around the heart very quickly, in a circular pattern, causing the heart to beat too fast." D) "For some reason, your electrical system is not on full charge, so they will have to put in new leads and a pacemaker to make it work better."

Ans: C

4. Which of the following patients will likely experience difficulty in maintaining lipoprotein synthesis resulting in elevated LDL levels? A) A 55-year-old male admitted for exacerbation of chronic obstructive pulmonary disease (COPD) B) A 44-year-old female admitted for hysterectomy due to cervical cancer with metastasis C) A 35-year-old patient with a history of hepatitis C and B with end-stage liver disease D) A 27-year-old patient with pancreatitis related to alcohol abuse

Ans: C

5. A 35-year-old female ultramarathon runner is admitted to the hospital following a day-long, 50-mile race because her urinary volume is drastically decreased and her urine is dark red. Tests indicate that she is in the initiating phase of acute tubular necrosis. Why is her urine red? A) Hematuria B) Hemoglobinuria C) Myoglobinuria D) Kidney bleeding

Ans: C

5. In which of the following hospital patients would the care team most realistically anticipate finding normal cholesterol levels? A) A 44-year-old male admitted for hyperglycemia and with a history of diabetic neuropathy B) A 77-year-old female admitted for rheumatoid arthritis exacerbation who is receiving hormone replacement therapy and with a history of hypothyroidism C) A 51-year-old male with a diagnosis of hemorrhagic stroke and consequent unilateral weakness D) A morbidly obese 50-year-old female who is taking diuretics and a beta-blocker to treat her hypertension

Ans: C

6. A number of older adults have come to attend a wellness clinic that includes both blood pressure monitoring and education about how to best control blood pressure. Which of the leader's following teaching points is most accurate? A) "It's important to minimize the amount of potassium and, especially, sodium in your diet." B) "High blood pressure is largely controllable, except for those with a significant family history or African Americans." C) "Too much alcohol, too little exercise, and too much body fat all contribute to high blood pressure." D) "Hypertension puts you at a significant risk of developing type 2 diabetes later in life."

Ans: C

6. During an automobile accident where the patient is bleeding heavily, which vascular component is the most distensible and can store large quantities of blood that can be returned to the circulation at this time of need? A) Liver and pancreas B) Kidneys C) Veins D) Aorta

Ans: C

6. Four patients were admitted to the emergency department with severe chest pain. All were given preliminary treatment with aspirin, morphine, oxygen, and nitrates and were monitored by ECG. Which patient most likely experienced myocardial infarction? A) A 33-year-old male whose pain started at 7 AM during moderate exercise and was relieved by nitrates; ECG was normal; cardiac markers remained stable. B) A 67-year-old female whose pain started at 2 AM while she was asleep and responded to nitrates; the ECG showed arrhythmias and ST-segment elevation; cardiac markers remained stable. C) An 80-year-old woman whose pain started at 6 AM shortly after awakening and was not relieved by nitrates or rest; the ECG showed ST-segment elevation with inverted T waves and abnormal Q waves; levels of cardiac markers subsequently rose. D) A 61-year-old man whose pain started at 9 AM during a short walk and responded to nitrates, but not to rest; ECG and cardiac markers remained stable, but anginal pattern worsened.

Ans: C

7. A family physician is providing care for a 61-year-old obese male who has a history of diabetes and hypertension. Blood work has indicated that the man has a GFR of 51 mL/minute with elevated serum creatinine levels. Which of the following statements will the physician most likely provide the client in light of these results? A) "We will regularly monitor your kidney function, but most likely your kidneys will be able to compensate on their own and intervention is not required." B) "You likely have chronic kidney disease, and there may be urine in your blood until it is controlled." C) "Your chronic kidney disease has likely been caused by your diabetes and high blood pressure." D) "You're in kidney failure, and I'll be starting dialysis treatment immediately."

Ans: C

7. A nurse educator is teaching a group of nurses at a long-term care facility about atrial fibrillation in light of its prevalence in older adults. Which of the following statements by the nurses would the educator most want to correct? A) "The electrical impulses go in chaotic directions, and so the atria can't contract properly." B) "An ECG of someone in atrial fibrillation would be almost random in appearance." C) "The contraction of the ventricles and the atria can range from 400 to 600 beats/minute." D) "It can be hard to measure at the bedside because not all ventricular beats make a palpable pulse."

Ans: C

7. Which of the following statements provides blood work results and rationale that would be most closely associated with acute coronary syndrome? A) Increased serum creatinine and troponin I as a result of enzyme release from damaged cells B) Increased serum potassium and decreased sodium as a result of myocardial cell lysis, release of normally intracellular potassium, and disruption of the sodium-potassium pump C) Elevated creatine kinase and troponin, both of which normally exist intracellularly rather than in circulation D) Low circulatory levels of myoglobin and creatine kinase as a result of the inflammatory response

Ans: C

8. A 66-year-old male presents to the emergency room accompanied by his wife who claims that he has been acting confused. The man is complaining of a sudden onset of severe weakness and malaise and has a dry cough and diarrhea. His temperature is 102.8°F, and his blood work indicates his sodium level at 126 mEq/L (normal 135 to 145 mEq/L). Based on this assessment, the nurse suspects the patient has A) bronchopneumonia. B) Mycoplasma pneumonia. C) Legionella pneumonia. D) pneumococcal pneumonia.

Ans: C

8. A pathologist is examining histological (tissue) samples from a client with an autoimmune disease. Which of the following characteristics of muscle samples would signal the pathologist that the samples are cardiac rather than skeletal muscle? A) The cell samples lack intercalated disks. B) The muscle cells have small and a few mitochondria. C) The cells have a poorly defined sarcoplasmic reticulum. D) The muscles are striated and composed of sarcomeres.

Ans: C

9. A health educator is performing a health promotion workshop with the staff of a large, urban homeless shelter, and a component of the teaching centers around tuberculosis. One of the staff members comments, "Anyone who's had contact with tuberculosis in the past can give it to any of the other residents of the shelter, even if they didn't get sick themselves." How could the educator best respond to this comment? A) "Many people do manage to fight off the infection, but you're right: they can still spread it by coughing or sneezing." B) "If someone has been previously exposed to tuberculosis, they are particularly infectious because they are often unaware of the disease." C) "Actually, people who have the latent form of the disease won't be sick and can't spread it either." D) "There isn't any real risk of them spreading it, but we would like to vaccinate everyone who's had any contact with it in the past."

Ans: C

9. In the early morning, an African American woman brings her 5-year-old son to the emergency room. The boy is wheezing, is short of breath, and has a dry cough. The mother states that he has always been very healthy. He went to bed with only a slight cold and a runny nose but woke her with his coughing shortly after 4 AM. His symptoms worsened so dramatically that she brought him to the hospital. The care team would most likely suspect that he has A) respiratory syncytial virus. B) influenza. C) asthma. D) pneumonia.

Ans: C

8. A male infant is brought into the clinic because of colic-like symptoms. The mother states he acts like something is hurting. After eating, he vomits most of the feeding and then assumes a fetal position. He is also not gaining weight. The nurse practitioner is thinking that he is displaying clinical manifestations of obstruction and may have which of the following medical diagnoses? A) Duodenal ulceration B) Constipation C) Pyloric stenosis D) Erosive esophagitis

Ans: C Feedback: An example of obstruction is hypertrophic pyloric stenosis, which can occur in infants with an abnormally thick muscularis layer in the terminal pylorus. A defect in the lining of the first part of the small intestine (duodenal ulcer) is usually caused by an infection with a bacterium (germ) called H. pylori. When food is ingested and digested but not excreted, it forms a blockage in the colon. Regular bowel movements are needed in order for this not to occur. When bowel movements are irregular, constipation may result. This infant appears to be vomiting his stomach content; therefore, no BM is occurring since no food is being digested in the small intestine. Gastrin provides the major stimulus for gastric acid production. Its action on the lower esophageal sphincter protects the esophageal mucosa when gastric acid levels are elevated. If stomach acids reflux into the esophagus, acid irritation and inflammation cause extensive injuries to the esophagus.

18. A male client complaining of chronic cramping, bloating, and diarrhea has been determined to have a deficiency in brush border enzymes within his small intestine. Which of the following meals or snacks high in carbohydrates and protein will likely exacerbate the client's signs and symptoms? A) Grapefruit and prunes B) Tossed salad with an oil and vinegar dressing C) Roast beef and a baked potato D) Tortilla chips and guacamole

Ans: C Feedback: Brush border enzymes are primarily responsible for the metabolism of carbohydrates and proteins, substances best exemplified by a baked potato and roast beef, respectively. Of the distracters, choice C has both high protein and high carbohydrate content. Distracters A, B, and D are not high in both carbohydrates and proteins.

16. Following a 14-day course of broad-spectrum antibiotics for the treatment of sepsis, a 60-year-old woman has developed watery diarrhea. Her care team attributes this to likely elimination of normal intestinal flora by the antibiotics. What other phenomena is most likely accompanying her low levels of normal flora? A) Decreased mineral and nutrient absorption; decreased carbohydrate metabolism B) Decreased pH of the stomach; increased pH of the lower GI tract C) Decreased fermentation of undigestible dietary residue; decreased vitamin absorption D) Proliferation of vitamin K; lower GI bleeding

Ans: C Feedback: Central among the functions of normal intestinal flora are the fermentation of dietary components that are not digestible and the facilitation of vitamin absorption. Mineral and nutrient absorption as well as carbohydrate metabolism are less likely to be affected, while vitamin K production would likely decrease somewhat. pH is unlikely to be affected.

1. A 55-year-old man has been diagnosed with a gastroesophageal reflux disease (GERD), in which the function of his lower esophageal sphincter is compromised. Which of the following consequences of this condition is most likely to occur? A) Decreased absorption of ingested foods and fluids B) Impaired control of the gastric emptying rate C) Protrusion of the stomach or regurgitation of stomach contents into the esophagus D) Inappropriate release of gastric enzymes

Ans: C Feedback: Given that the role of the lower esophageal sphincter is to control the exchange of foods and fluids, a deficit is likely to allow the stomach contents, or the stomach itself, to protrude into the esophagus. Absorption is unlikely to be directly affected, and the sphincter is not responsible for controlling gastric emptying or enzyme secretion.

7. A speech therapist is performing a swallowing assessment on a 72-year-old man who has suffered a stroke 3 weeks ago. The man has been NPO (nothing by mouth) since his stroke, and the health care team is considering the introduction of oral food. The speech therapist is cueing the client to swallow to preclude either aspiration of food or pocketing of food in the sides of his mouth. The client most likely to have conscious control over which of the following processes listed below involved in swallowing? A) Initiation of primary peristalsis B) Moving the epiglottis back to cover the larynx C) Moving a bolus to the posterior wall of the pharynx D) Moving the bolus backward in the esophagus

Ans: C Feedback: Moving a bolus to the posterior wall of the pharynx is a component of the oral, or voluntary, phase of swallowing. Initiation of primary peristalsis, moving the epiglottis back to cover the larynx, and moving the bolus backward in the esophagus are all involuntary components of the pharyngeal and esophageal phases of swallowing.

12. A science teacher is talking to a group of fifth graders about the role of "spit." During the course of the discussion, the teacher asks the students which of the following are functions of saliva? A) Good protection device if someone is being attacked B) Will have more saliva production when anxious, such as right before a test C) Has antibacterial action to help keep the mouth clean D) Secretes acid to chemically break down fatty foods like French fries E) Secretes the enzyme pepsin

Ans: C Feedback: Saliva has three functions. The first is to protect and lubricate. The second is to provide antimicrobial protection. The third is to initiate digestion of starches by secreting enzymes. If anxious, SNS activity causes dry mouth. Saliva begins breaking down starches (carbohydrates), not fatty foods. The chief cells secrete pepsinogen, which is converted into pepsin, the enzyme that breaks down protein. This occurs in the stomach.

4. The instructor asks a group of nursing students to explain the function of the omentum. The students will respond based on which pathophysiologic principle? A) It holds organs in place. B) It attaches the jejunum and ileum to the abdominal wall. C) It has lots of mobility and moves around in the peritoneal cavity with peristaltic movements. D) It is mainly there to prevent any noxious substance from inner into the gut.

Ans: C Feedback: The greater omentum has considerable mobility and moves around in the peritoneal cavity with the peristaltic movements of the intestines. It also cushions the abdominal organs against injury and provides insulation against the loss of body heat. The mesentery holds the organs in place and attaches the jejunum and ileum to the abdominal wall. The mucosal layer acts as a barrier to prevent the entry of noxious substances and pathogenic organisms.

17. A patient has just been diagnosed with pernicious anemia. The patient asks the nurse why his body is not able to absorb vitamin B12. The nurse responds, A) "You have too many ulcers in your stomach." B) "More than likely, you were born with deficient vitamin B12 stores." C) "Your stomach is not secreting a substance known as intrinsic factor, which is needed to absorb vitamin B12." D) "Your daily intake of high saturated fats is interfering with the stomach's ability to absorb the nutrients that it needs, especially vitamin B12."

Ans: C Feedback: Vitamin B12 is not absorbed in the absence of intrinsic factor, which is secreted by the parietal cells of the stomach. Ulcers in the stomach do not cause pernicious anemia. This is not a congenital problem, nor does the intake of fats interfere with the secretion of intrinsic factor.

1. A 66-year-old obese man with diagnoses of ischemic heart disease has been diagnosed with heart failure that his care team has characterized as attributable to systolic dysfunction. Which of the following assessment findings is inconsistent with his diagnosis? A) His resting blood pressure is normally in the range of 150/90, and an echocardiogram indicates his ejection fraction is 30%. B) His end-diastolic volume is higher than normal, and his resting heart rate is regular and 82 beats/minute. C) He is presently volume overloaded following several days of intravenous fluid replacement. D) Ventricular dilation and wall tension are significantly lower than normal.

Ans: D

11. A nurse is providing care for a client who has a history of severe atherosclerosis. Which of the following clinical manifestations of the client's illness should the nurse anticipate and assess in the client? A) Motor deficits in muscles distal to plaque formation B) Peripheral vasodilation to compensate for ischemia C) Cognitive deficits due to ischemia or thrombosis D) Aneurysm formation due to weakening of blood vessel walls E) Necrosis of the vessel wall

Ans: D

11. Around 3 weeks after razing an old chicken house, a 71-year-old retired farmer has developed a fever, nausea, and vomiting. After ruling out more common health problems, his care provider eventually made a diagnosis of histoplasmosis. Which of the following processes is most likely taking place? A) Toxin production by Histoplasma capsulatum is triggering an immune response. B) Antibody production against the offending fungi is delayed by the patient's age and the virulence of the organism. C) Spore inhalation initiates an autoimmune response that produces the associated symptoms. D) Macrophages are able to remove the offending fungi from the bloodstream but can't destroy them.

Ans: D

12. A 24-year-old woman undergoing a premarital screening test is found to have elevated levels of AST, ALT, and IgG, but no antibody-specific markers for viral hepatitis. A liver biopsy reveals inflammation and cellular damage. Which of the following treatments is most likely to be effective for her? A) Lamivudine B) Peginterferon and ribavirin C) Interferon alfa-2b D) Corticosteroids and immunosuppressant drugs

Ans: D

13. A 51-year-old male professional is in the habit of consuming six to eight rum and cokes each evening after work. He assures the nurse practitioner who is performing his regular physical exam that his drinking is under control and does not have negative implications for his work or family life. How could the nurse best respond to the client's statement? A) "You are more than likely inflicting damage on your liver, but this damage would cease as soon as you quit drinking." B) "That may be the case, but you are still creating a high risk of hepatitis A or B or liver cancer." C) "In spite of that, the amount of alcohol you are drinking is likely to result first in cirrhosis and, if you continue, in hepatitis or fatty liver changes." D) "When your body has to regularly break down that much alcohol, your blood and the functional cells in your liver accumulate a lot of potentially damaging toxic byproducts."

Ans: D

13. A nurse is using a stethoscope and blood pressure cuff to manually measure a client's blood pressure. The nurse knows that which of the following facts related to blood flow underlies the ability to hear blood pressure by auscultation (listening)? A) The force of blood with each cardiac contraction produces friction on vessel walls that can be heard and felt. B) The movement of smooth muscle surrounding vessels produces noise that is audible by a stethoscope. C) Turbulent flow of blood during systole produces sound while laminar flow during diastole is silent. D) Pressure pulsation that exceeds the velocity of blood flow is audible and coincides with systolic BP.

Ans: D

13. A patient has just been diagnosed with acute glomerulonephritis. Which question should the nurse ask this client in attempting to establish a cause? A) "Do you have a history of heart failure?" B) "Have you recently had kidney stones?" C) "Have you ever been diagnosed with diabetes?" D) "Have you had any type of infection within the last 2 weeks?"

Ans: D

14. A 31-year-old woman with a congenital heart defect reports episodes of light-headedness and syncope, with occasional palpitations. A resting electrocardiogram reveals sinus bradycardia, and she is suspected to have sick sinus syndrome. Which of the following diagnostic methods is the best choice to investigate the suspicion? A) Signal-averaged ECG B) Exercise stress testing C) Electrophysiologic study D) Holter monitoring

Ans: D

15. A young woman has been diagnosed by her family physician with primary Raynaud disease. The woman is distraught stating, "I've always been healthy, and I can't believe I have a disease now." What would be her physician's most appropriate response? A) "This likely won't have a huge effect on your quality of life, and I'll prescribe anticlotting drugs to prevent attacks." B) "I'll teach you some strategies to minimize its effect on your life, and minor surgery to open up your blood vessels will help too." C) "You need to make sure you never start smoking, and most of the symptoms can be alleviated by regular physical activity." D) "If you make sure to keep yourself warm, it will have a fairly minimal effect; I'll also give you pills to enhance your circulation."

Ans: D

17. A 42-year-old male has been diagnosed with renal failure secondary to diabetes mellitus and is scheduled to begin dialysis soon. Which of the following statements by the client reflects an accurate understanding of the process of hemodialysis? A) "It's stressful knowing that committing to dialysis means I can't qualify for a kidney transplant." B) "I know I'll have to go to a hospital or dialysis center for treatment." C) "Changing my schedule to accommodate 3 or 4 hours of hemodialysis each day will be difficult." D) "I won't be able to go about my normal routine during treatment."

Ans: D

17. Which of the following situations would be most deserving of a pediatrician's attention? A) The mother of an infant 2 days postpartum notes that her baby has intermittent periods of hyperventilation followed by slow respirations or even brief periods of apnea. B) A volunteer in the nursery notes that one of the infants, aged 2 weeks, appears unable to breathe through his mouth, even when his nose is congested. C) A neonate is visibly flaring her nostrils on inspiration. D) A midwife notes that a newborn infant's chest is retracting on inspiration and that the child is grunting.

Ans: D

18. A 70-year-old woman with ongoing severe atrial fibrillation is scheduled for defibrillation. What is an aspect of the rationale and physiology of defibrillation treatment? A) Interruption of disorganized impulses by the current allows the AV node to readopt its normal pacemaker role. B) Defibrillation can be achieved using either a transcutaneous or transvenous pacemaker. C) Defibrillation must be coincided with the R wave of the ECG in order to be successful. D) The goal is to depolarize the entire heart during the passage of current.

Ans: D

19. A physiotherapist is measuring the lying, sitting, and standing blood pressure of a patient who has been admitted to hospital following a syncopal episode and recent falls. Which of the following facts about the patient best relates to these health problems? A) The patient is male and has a history of hypertension. B) The patient's cardiac ejection fraction was 40% during his last echocardiogram. C) The patient has a history of acute and chronic renal failure. D) The client is 89 years old and takes a diuretic medication for his congestive heart failure.

Ans: D

19. While on tour, a 32-year-old male musician has presented to the emergency department of a hospital after a concert complaining of severe and sudden abdominal pain. He admits to a history of copious alcohol use in recent years, and his vital signs include temperature 46.8°C (101.8°F), blood pressure 89/48 mm Hg, and heart rate 116 beats/minute. Blood work indicates that his serum levels of C-reactive protein, amylase, and lipase are all elevated. Which of the following diagnoses would the care team suspect first? A) Hepatitis C B) Cholecystitis C) Liver cirrhosis D) Acute pancreatitis

Ans: D

2. A nursing instructor is explaining the role of vascular smooth muscle cells in relation to increases in systemic circulation. During discussion, which neurotransmitter is primarily responsible for contraction of the entire muscle cell layer thus resulting in decreased vessel lumen radius? A) Nitric oxide B) Adrenal glands C) Fibroblast growth factor D) Norepinephrine

Ans: D

20. Which of the following situations related to transition from fetal to perinatal circulation would be most likely to necessitate medical intervention? A) Pressure in pulmonary circulation and the right side of the infant's heart fall markedly. B) Alveolar oxygen tension increases causing reversal of pulmonary vasoconstriction of the fetal arteries. C) Systemic vascular resistance and left ventricular pressure are both increasing. D) Pulmonary vascular resistance, related to muscle regression in the pulmonary arteries, rises over the course of the infant's first week.

Ans: D

20. While rock climbing, a 22-year-old male has endured a severe head injury. Which of the following statements best captures expected clinical manifestations and treatments for his immediate condition? A) Oxygen therapy is likely to decrease his respiratory drive and produce an increase in PCO2. B) Cheyne-Stokes breathing is likely but will respond to bronchodilators. C) The client is unlikely to respond to supplementary oxygen therapy due to impaired diffusion. D) Hypoventilation may exist, resulting in increased PCO2 and hypoxemia that may require mechanical ventilation.

Ans: D

3. An 81-year-old male resident of a long-term care facility has a long-standing diagnosis of heart failure. Which of the following short-term and longer-term compensatory mechanisms are least likely to decrease the symptoms of his heart failure? A) An increase in preload via the Frank-Starling mechanism B) Sympathetic stimulation and increased serum levels of epinephrine and norepinephrine C) Activation of the renin-angiotensin-aldosterone system and secretion of brain natriuretic peptide (BNP) D) AV node pacemaking activity and vagal nerve suppression

Ans: D

4. A 48-year-old woman has been diagnosed with extrahepatic cholestasis following a thorough history, ultrasound, and blood work. Which of the following symptoms most likely caused her to seek medical treatment, and what consequence to her health problem would the medical team anticipate? A) Complaints of lower flank pain with consequences of impaired fat metabolism B) Anorexia with consequences of impaired drug metabolism and blood filtration C) Skin xanthomas (focal accumulations of cholesterol) with consequences of increased risk of bleeding due to deficient clotting factors D) Pruritus with consequences of deficient levels of fat-soluble vitamins

Ans: D

4. A 54-year-old man with a long-standing diagnosis of essential hypertension is meeting with his physician. The patient's physician would anticipate that which of the following phenomena is most likely occurring? A) The patient's juxtaglomerular cells are releasing aldosterone as a result of sympathetic stimulation. B) Epinephrine from his adrenal gland is initiating the renin-angiotensin-aldosterone system. C) Vasopressin is exerting an effect on his chemoreceptors and baroreceptors resulting in vasoconstriction. D) The conversion of angiotensin I to angiotensin II in his lungs causes increases in blood pressure and sodium reabsorption.

Ans: D

5. A physician is teaching a group of medical students about the physiological basis for damage to the circulatory and neurological systems that can accompany hypotension. Which of the following responses by a student would warrant correction by the physician? A) "As vessel wall thickness increases, tension decreases." B) "Smaller blood vessels require more pressure to overcome wall tension." C) "The smaller the vessel radius, the greater the pressure needed to keep it open." D) "Tension and vessel thickness increase proportionately."

Ans: D

6. A nurse practitioner is instructing a group of older adults about the risks associated with high cholesterol. Which of the following teaching points should the participants try to integrate into their lifestyle after the teaching session? A) "Remember the 'H' in HDL and the 'L' in LDL correspond to high danger and low danger to your health." B) "Having high cholesterol increases your risk of developing diabetes and irregular heart rate." C) "Smoking and being overweight increases your risk of primary hypercholesterolemia." D) "Your family history of hypercholesterolemia is important, but there are things you can do to compensate for a high inherited risk."

Ans: D

7. A client has suffered damage to his pericardium following a motor vehicle accident. Which consequence could be a possible complication of damaged pericardium that his care providers should assess for? A) Impaired physical restraint of the left ventricle B) Increased friction during the contraction/relaxation cycle C) Reduced protection from infectious organisms D) Impaired regulation of myocardial contraction

Ans: D

7. Mr. V. has been admitted for exacerbation of his chronic heart failure (HF). When the nurse walks into his room, he is sitting on the edge of the bed, gasping for air, and his lips are dusty blue. Vital signs reveal heart rate of 112, respiratory rate of 36, and pulse oximeter reading of 81%. He starts coughing up frothy pink sputum. The priority intervention is to A) have medical supply department bring up suction equipment. B) apply oxygen via nasal cannula at 3 lpm. C) page the respiratory therapist to come give him a breathing treatment. D) call for emergency assistance utilizing hospital protocol.

Ans: D

7. Which of the following statements best captures the etiology of the acute response phase of extrinsic (atopic) asthma? A) IgG production is heightened as a consequence of exposure to an allergen. B) Airway remodeling results in airflow limitations. C) Epithelial injury and edema occur along with changes in mucociliary function. D) Chemical mediators are released from presensitized mast cells.

Ans: D

8. A number of clients have presented to the emergency department in the last 32 hours with complaints that are preliminarily indicative of myocardial infarction. Which of the following clients is least likely to have an ST-segment myocardial infarction (STEMI)? A) A 70-year-old woman who is complaining of shortness of breath and vague chest discomfort B) A 66-year-old man who has presented with fatigue, nausea and vomiting, and cool, moist skin C) A 43-year-old man who woke up with substernal pain that is radiating to his neck and jaw D) A 71-year-old man who has moist skin, fever, and chest pain that is excruciating when he moves but relieved when at rest

Ans: D

8. When a 55-year-old patient's routine blood work returns, the nurse notes that his C-reactive protein (CRP) is elevated. The patient asks what that means. The nurse responds, A) "You must eat a lot of red meat since this means you have a lot of fat floating in your vessels." B) "You are consuming high levels of folate, which works with the B vitamins and riboflavin to metabolize animal protein." C) "This means you have high levels of HDL to balance the LDL found in animal proteins." D) "This means you have elevated serum markers for systemic inflammation that has been associated with vascular disease."

Ans: D

9. A 63-year-old male client has been diagnosed with a bundle branch block. How will this client's care team most likely expect his condition to be expressed diagnostically? A) His AV node will be performing the primary pacemaker role due to inadequacy of the SA node. B) His ECG will show a flattened P wave as a result of impaired atrial depolarization. C) Conduction from the Purkinje fibers to the bundle branches is compromised by inadequate conduction. D) His ECG will show an inordinately wide QRS complex because impulses are bypassing the normal conduction tissue.

Ans: D

9. An autopsy is being performed on a 44-year-old female who died unexpectedly of heart failure. Which of the following components of the pathologist's report is most suggestive of a possible history of poorly controlled blood pressure? A) "Scarring of the urethra suggestive of recurrent urinary tract infections is evident." B) "Bilateral renal hypertrophy is noted." C) "Vessel wall changes suggestive of venous stasis are evident." D) "Arterial sclerosis of subcortical brain regions is noted."

Ans: D

9. Which of the following individuals is at the highest risk of developing a urinary tract infection (UTI)? A) A 60-year-old man with a history of cardiovascular disease who is recovering in hospital from a coronary artery bypass graft B) A 66-year-old man undergoing dialysis for the treatment of chronic renal failure secondary to hypertension C) A 38-year-old man with high urine output due to antidiuretic hormone insufficiency D) A 30-year-old woman with poorly controlled diabetes mellitus

Ans: D

14. A 77-year-old lifetime smoker has been diagnosed with a tumor in his lung at the site of an old tubercle scarring site, located in a peripheral area of his bronchiolar tissue. What is this client's most likely diagnosis? A) Squamous cell carcinoma B) Small cell lung cancer C) Large cell carcinoma D) Adenocarcinoma

Ans: D .

9. A 79-year-old male resident of a long-term care facility has contracted Clostridium difficile and is experiencing consequent diarrhea. Auscultation of the man's abdomen indicates hyperactive bowel sounds. What process in the man's small intestine is most likely accompanying his current status? A) Pathogenic microorganisms are causing dilation of his small intestine, increasing motility. B) Segmentation waves have become more frequent as a result of his infection. C) Intestinal stasis brought on by infection is preventing his small intestine from sufficiently slowing the rate of motility. D) Inflammation is accompanied by an increase in peristaltic movements of his small intestine.

Ans: D Feedback: Inflammation of the small intestine is accompanied by an increase in motility, an effect that is the result of increased peristaltic waves. Segmentation waves are responsible primarily for mixing rather than moving food. Neither dilation nor inadequate slowing of passage contents is responsible for the increase in motility.

14. A 77-year-old woman has been admitted to hospital following several weeks of increasing fatigue. On observation, she is pale, and blood work indicates she has low hemoglobin and red cell counts. Stool tests for occult blood are positive, and following endoscopy, she has been diagnosed with an upper GI bleed that has been shown to originate in her stomach. She admits to regularly exceeding the recommended doses of nonsteroidal anti-inflammatory drugs (NSAIDs) in an effort to control her rheumatoid arthritis. Which of the following phenomena is most likely responsible for her present health problems? A) NSAIDs increase the gastric production of gastrin, increasing gastric secretions and lowering stomach pH. B) Drugs such as NSAIDs increase the H+ levels and thus decrease gastric pH, resulting in insult to the stomach lining. C) NSAIDs, aspirin, and other drugs increase prostaglandin synthesis, resulting in disruption of cellular structures lining the stomach. D) NSAIDs can disrupt the permeability of the gastric mucosa, causing hydrogen ions to accumulate in the mucosal cells of the lining.

Ans: D Feedback: NSAIDs damage the mucosal barrier, allowing hydrogen ions to damage cells of the stomach lining. They do not directly increase gastrin production or H+ levels, and decreased, not increased, prostaglandin synthesis would potentially compromise the gastric surfaces.

10. Which of the following statements best communicates the nature of movements in the colon? A) The internal and external anal sphincters control the movement of feces into and through the colon. B) Haustrations move the colon contents along with 2- to 3-minute rest periods between movements. C) Segmentation waves ensure that all surfaces of the feces are exposed to the colon surface. D) Large segments of the colon contract as a unit for around 30 seconds.

Ans: D Feedback: The colon, unlike the small intestine, moves contents by the simultaneous contraction of large segments. The anal sphincters control defecation rather than movements within the colon, and haustrations perform mixing, not motility. Segmentation waves are present in the small intestine.

16. When explaining to a patient why he only had minimal muscle damage following 99% occlusion of the left anterior descending artery, the nurse will explain this is primarily due to A) the possibility that the person has elevated INR levels that prevent blood from backlogging in the vessel. B) development of collateral circulation that builds channels between some of the smaller arteries usually when the flow is decreased gradually. C) good genetic makeup that allows occluded arteries to keep vasodilating to meet metabolic needs. D) the release of substances formed by special glands that tr

Ansport the blood cell-by-cell through smaller spaces. ++Ans: B

4. A member of the health care team is researching the etiology and pathogenesis of a number of clients who are under his care in a hospital context. Which of the following aspects of clients' situations best characterizes pathogenesis rather than etiology? A) A client who has been exposed to the Mycobacterium tuberculosis bacterium B) A client who has increasing serum ammonia levels due to liver cirrhosis C) A client who was admitted with the effects of methyl alcohol poisoning D) A client with multiple skeletal injuries secondary to a motor vehicle accident

B) A client who has increasing serum ammonia levels due to liver cirrhosis

9. The laboratory technologists are discussing a new blood test that helps establish a differential diagnosis between shortness of breath with a cardiac etiology and shortness of breath with a respiratory/pulmonary etiology. A positive result is purported to indicate a cardiac etiology. The marketers of the test report that 99.8% of patients who have confirmed cardiac etiologies test positive in the test. However, 1.3% of patients who do not have cardiac etiologies for their shortness of breath also test positive. Which of the following statements best characterizes this blood test? A) Low validity; high reliability B) High sensitivity; low specificity C) High specificity; low reliability D) High sensitivity; low reliability

B) High sensitivity; low specificity

3. A male, lifetime smoker has died because of chronic obstructive pulmonary disease. Which of the following phenomena regarding his alveoli would his care team expect in the weeks prior to his death? A) Proliferation of natural killer (NK) cells in the alveolar lumen B) Large numbers of alveolar macrophages in septal connective tissue C) The presence of tubercles in the interalveolar spaces D) Compensatory regeneration of type I alveolar cells

B) Large numbers of alveolar macrophages in septal connective tissue

4. Reviewing pathology for an exam on pulmonary vasculature, the nursing student states that blood enters the right side of the heart via the vena cavae, then to the right atrium, right ventricle, and then which vessel carries the deoxygenated blood into the pulmonary system? A) Pulmonary capillaries B) Pulmonary artery C) Pulmonary vein D) Ductus arteriosus

B) Pulmonary artery

2. A community health nurse is teaching a group of recent graduates about the large variety of factors that influence an individual's health or lack thereof. The nurse is referring to the Healthy People 2020 report from the U.S. Department of Health and Human Services as a teaching example. Of the following aspects discussed, which would be considered a determinant of health that is outside the focus of this report? A) The client has a diverse background by being of Asian and Native American descent and practices various alternative therapies to minimize effects of stress. B) The client has a family history of cardiovascular disease related to hypercholesterolemia and remains noncompliant with the treatment regime. C) The client has a good career with exceptional preventative health care benefits. D) The client lives in an affluent, clean, suburban community with access to many health care facilities.

B) The client has a family history of cardiovascular disease related to hypercholesterolemia and remains noncompliant with the treatment regime.

6. A 77-year-old man is a hospital inpatient admitted for exacerbation of his chronic obstructive pulmonary disease (COPD), and a respiratory therapist (RT) is assessing the client for the first time. Which of the following aspects of the patient's current state of health would be best characterized as a symptom rather than a sign? A) The patient's oxygen saturation is 83% by pulse oxymetry. B) The patient notes that he has increased work of breathing when lying supine. C) The RT hears diminished breath sounds to the patient's lower lung fields bilaterally. D) The patient's respiratory rate is 31 breaths/minute.

B) The patient notes that he has increased work of breathing when lying supine.

17. Which of the following situations is most likely to result in an increased binding affinity of hemoglobin for oxygen? A) A client is in respiratory acidosis, with a low pH. B) Three of four binding sites on a client's hemoglobin molecule are occupied by oxygen. C) A client's body temperature is elevated as a result of an infectious process. D) An increase in 2,3-diphosphoglycerate enhances the loading of oxygen.

B) Three of four binding sites on a client's hemoglobin molecule are occupied by oxygen.

20. A nurse in an acute medical unit is providing care for a number of patients with a variety of diagnoses. Which of the following patients most likely exhibits risk factors for impaired coughing? A patient with A) an injury to her cerebellum. B) a nasogastric (NG) tube attached to suction. C) a diagnosis of viral pneumonia. D) diagnosis of diabetes mellitus and morbid obese.

B) a nasogastric (NG) tube attached to suction.

10. While working in the newborn ICU, the nurses receive a call that an infant, gestational age of 23 weeks, is being air flighted to the level 3 trauma nursery. The priority intervention for this infant would be A) insertion of an umbilical line for fluids. B) intubation and mechanical ventilation. C) insertion of a feeding tube. D) insertion of an intraventricular catheter.

B) intubation and mechanical ventilation.

5. The nurse is hearing diminished breath sounds and a "grating" sound during respirations. This is consistent with excess collection of fluid in the pleural cavity. The medical term for this is A) pleurisy. B) pleural effusion. C) pneumothorax. D) poor lung compliance.

B) pleural effusion.

5. A new myocardial infarction patient requiring angioplasty and stent placement has arrived to his first cardiac rehabilitation appointment. In this first session, a review of the pathogenesis of coronary artery disease is addressed. Which statement by the patient verifies to the nurse that he has understood the nurse's teachings about coronary artery disease? A) "All I have to do is stop smoking, and then I won't have any more heart attacks." B) "My artery was clogged by fat, so I will need to stop eating fatty foods like French fries every day." C) "Sounds like this began because of inflammation inside my artery that made it easy to form fatty streaks, which lead to my clogged artery." D) "If I do not exercise regularly to get my heart rate up, blood pools in the veins causing a clot that stops blood flow to the muscle, and I will have a heart attack."

C) "Sounds like this began because of inflammation inside my artery that made it easy to form fatty streaks, which lead to my clogged artery."

19. Which of the following neurological patients is most likely to have abnormalities in breathing regulation? A) A 23-year-old male who has an injury to his frontal lobe following a sports injury B) A 45-year-old female with a spinal cord injury at C7 following a motor vehicle accident C) A 34-year-old male with damage to his upper and lower pons following a blow to the back of the head D) A 66-year-old male with temporal lobe infarcts secondary to a stroke

C) A 34-year-old male with damage to his upper and lower pons following a blow to the back of the head

14. An epidemiologist is conducting a program of research aimed at identifying factors associated with incidence and prevalence of congenital cardiac defects in infants. The researcher has recruited a large number of mothers whose infants were born with cardiac defects as well as mothers whose infants were born with healthy hearts. The researcher is comparing the nutritional habits of all the mothers while their babies were in utero. Which of the following types of study is the epidemiologist most likely conducting? A) Cohort study B) Cross-sectional study C) Case-control study D) Risk factor study

C) Case-control study

18. A nurse in a respiratory unit of a hospital is providing care for a client with end-stage lung disease. Consequently, measurement of the client's arterial blood gases indicates increased PCO2. Which of the following associated consequences would the nurse anticipate? A) A shift to the left of the oxygen-hemoglobin dissociation curve B) Lower than normal production of HCO3 C) Higher than normal production of H+ D) An absence of carbaminohemoglobin

C) Higher than normal production of H+

1. As a result of dehydration, a client's epithelial cells are producing insufficient amounts of mucus. Consequently, the client's mucociliary blanket is compromised. Which of the following changes would the care provider anticipate as a direct result of this change? A) Impaired function of the client's cilia B) Decreased levels of oxygen saturation C) Increased amounts of bacteria in the lungs D) Increased carbon dioxide levels

C) Increased amounts of bacteria in the lungs

8. A client who presented with shortness of breath and difficulty climbing stairs has been diagnosed with pulmonary fibrosis, a disease characterized by scarring of the alveoli. Upon assessment of the lungs, what clinical manifestations should the nurse expect? A) Rapid, deep breaths B) Wheezing throughout lung fields C) Short, shallow breaths D) Pursed-lip breaths with slow, steady breaths

C) Short, shallow breaths

17. A multidisciplinary health care team operates a program aimed at the prevention, identification, and treatment of diabetes on a large Indian reservation. Which of the following aspects of the program would be most likely to be classified as secondary prevention? A) Regularly scheduled wound dressing changes for clients who have foot ulcers secondary to peripheral neuropathy and impaired wound healing B) Teaching school children how a nutritious, traditional diet can lessen their chances of developing adult-onset diabetes C) Staffing a booth where community residents who are attending a baseball tournament can have their blood glucose levels checked D) Administering oral antihyperglycemic medications to clients who have a diagnosis of diabetes

C) Staffing a booth where community residents who are attending a baseball tournament can have their blood glucose levels checked

18. An occupational therapist conducts a group therapy program called MindWorks with older adults who have diagnoses of dementia and Alzheimer disease. The goal of the group is to slow the cognitive decline of clients by engaging them in regular, organized mental activity such as reading maps and solving puzzles. How would the program most likely be characterized? A) Primary prevention B) Secondary prevention C) Tertiary prevention D) Prognosis enhancement

C) Tertiary prevention

1. As part of a public health initiative, a nurse is teaching a group of older adults about ways to promote and maintain their health. Recognizing that the common cold is a frequent source of ailment, the nurse is addressing this health problem. Which of the following teaching points about the common cold is most accurate? A) "You shouldn't be taking antibiotics for a cold until your doctor has confirmed exactly which bug is causing your cold." B) "It's important to both cover your mouth when you cough or sneeze and encourage others to do so, since most colds are spread by inhaling the germs." C) "Scientists don't yet know exactly what virus causes the cold, and there is not likely to be a vaccine until this is known." D) "Use caution when choosing over-the-counter drugs for your cold; most people do best with rest and antifever medications."

D) "Use caution when choosing over-the-counter drugs for your cold; most people do best with rest and antifever medications.

3. A physician is providing care for a number of patients on a medical unit of a large, university hospital. The physician is discussing with a colleague the differentiation between diseases that are caused by abnormal molecules and diseases that cause disease. Which of the following patients most clearly demonstrates the consequences of molecules that cause disease? A) A 31-year-old woman with sickle cell anemia who is receiving a transfusion of packed red blood cells B) A 91-year-old woman who has experienced an ischemic stroke resulting from familial hypercholesterolemia C) A 19-year-old man with exacerbation of his cystic fibrosis requiring oxygen therapy and chest physiotherapy D) A 30-year-old homeless man who has Pneumocystis carinii pneumonia (PCP) and is HIV positive.

D) A 30-year-old homeless man who has Pneumocystis carinii pneumonia (PCP) and is HIV positive.

14. Due to complications, a male postoperative patient has been unable to mobilize secretions for several days following surgery and develops atelectasis. Which of the following processes would his care team anticipate with relation to his health problem? A) Vasodilation in the alveolar vessels in the affected region of his lung B) Increased workload for the left side of the patient's heart C) Increased blood flow to the area of atelectasis D) Directing blood flow away from the lung regions that are hypoxic

D) Directing blood flow away from the lung regions that are hypoxic

9. A female patient is requiring supplementary oxygen by face mask due to her reduced lung compliance. Which of the following pathophysiological processes is most likely a contributor to her low lung compliance? A) The woman's lungs have more recoil than a healthy person's. B) Her type II alveolar cells are producing a slight excess of surfactant. C) Turbulent airflow is taking place in the patient's large airways. D) Her thoracic cage is less flexible than when she was healthy.

D) Her thoracic cage is less flexible than when she was healthy.

13. A particular disease has a debilitating effect on the ability of sufferers to perform their activities of daily living and is a significant cause of decreased quality of life. However, few people die as a result of the disease's direct effects. There are hundreds of thousands of Americans living with the disease but relatively few new cases in recent years. Which of the following statements best conveys an accurate epidemiological characterization of the disease? A) Low mortality; high morbidity; low prevalence; high incidence B) Low mortality; high morbidity; high incidence; low prevalence C) High mortality; low morbidity; high incidence; low prevalence D) High morbidity; low mortality; high prevalence, low incidence

D) High morbidity; low mortality; high prevalence, low incidence

6. Which of the following statements best conveys an aspect of the respiratory pressures that govern ventilation? A) Intrapleural pressure slightly exceeds that of the inflated lung. B) The chest wall exerts positive pressure on the lungs that contributes to expiration. C) The lungs are prevented from collapsing by constant positive intrapulmonary pressure. D) Negative intrapleural pressure holds the lungs against the chest wall.

D) Negative intrapleural pressure holds the lungs against the chest wall.

19. The clinical educator of a hospital medical unit has the mandate of establishing evidence-based practice guidelines for the nursing care on the unit. Which of the following statements most accurately captures a guiding principle of the nurse's task? A) Evidence-based practice guidelines will be rooted in research rather than nurses' subjective practice preferences and experiences. B) Guidelines are synonymous with systematic research reviews. C) The need for continuity and standardization of guidelines will mean that they will be fixed rather than changeable. D) The guidelines will combine individual expertise with external systematic evidence.

D) The guidelines will combine individual expertise with external systematic evidence.

13. The physician mentions the patient has developed alveolar dead space. The nurse recognizes that this means A) air that is moved in and out of the lungs with each breath. B) air that cannot participate in gas exchange and remains in the main bronchus. C) air is trapped in the conducting airways. D) alveoli are ventilated but not perfused.

D) alveoli are ventilated but not perfused.

12. As of November 1, 2012, there were a total of 10 confirmed cases of Hantavirus infection in people who were recent visitors (mid-June to end of August, 2012) to Yosemite National Park. Three visitors with confirmed cases died. Health officials believe that 9 out of the 10 people with Hantavirus were exposed while staying in Curry Village in the Signature Tent Cabins. This is an example of A) what the anticipated mortality rate would be if a family of five were planning to vacation in Yosemite National Park. B) the prevalence of Hantavirus one can anticipate if he or she is going to vacation in Yosemite National Park. C) the low rate of morbidity one can expect while traveling to Yosemite National Park. D) the incidence of people who are at risk for developing Hantavirus while staying in Yosemite National Park.

D) the incidence of people who are at risk for developing Hantavirus while staying in Yosemite National Park.


Conjuntos de estudio relacionados

De Paola - Arch Hist 2 exam 1 study guide

View Set

MED SURG PREP U 31, Ch. 31 Assessment and Management of Patients with Hypertension

View Set

World History Midterm Questions (Unit 1 and 2 ); CSU

View Set

Sole Proprietorship, Partnerships

View Set

The Outsiders Ch. 3-4 Vocabulary

View Set

American History Exam - Past Tests

View Set

ABA- Self- Monitoring and Management

View Set

MKT 3013 Sample Quiz & Test, Ch 4 (Exam 2)

View Set

ELECTRICAL 2 FINAL (quiz/exam questions)

View Set

ATI: Comprehensive physical assessment of an adult

View Set

Język niemiecki - HAUSTIERE (cz. 2)

View Set

Macroeconomics practice questions Ch.1

View Set